Nursing 404 OB Exam 1 Nclex Question

Pataasin ang iyong marka sa homework at exams ngayon gamit ang Quizwiz!

Which of the following findings during the fourth stage would require immediate interventions by the nurse? a. Fundus firm and at midline b. Fundus firm, deviated to the right, with slight distention over the symphysis pubis c. Blood pressure and pulse slightly lower than reading during second stage of labor d. Lochia is bright red, with a few small clots

B Even though the fundus is firm, it is not midline and the bladder is filling. A full bladder will interfere with contractions of the uterus and lead to increased bleeding. The rest of the answer choices are within normal limits for this stage.

A multigravida at 37 weeks of gestation is admitted to the labor room. She has contractions every 3 to 4 minutes lasting 40 to 50 seconds and no history of clear fluid leakage from the vagina, but complains of bright red bleeding for the past hour. The fetal heart rate is 145 beats/min (bpm). What should be the nurse's next intervention? a. Call the physician. b. Perform a vaginal exam to determine imminence of birth. c. Continue to monitor contractions and fetal heart rate. d. Administer an enema according to protocol of the agency.

A Bright red bleeding is a sign of complications, and the physician or primary health care provider should be notified immediately. Vaginal exams or enemas are contraindicated in the presence of bleeding. Continuing to monitor the mother and fetus is important after notifying the health care provider.

During a vaginal exam, the physician stimulates the fetal scalp. The fetal heart rate accelerated from 140 to 155 bpm for about 30 seconds. The nurse should: a. Record this reassuring fetal reaction. b. Notify the physician because this reaction is nonreassuring. c. Assist the woman into a side-lying position. d. Administer oxygen at 8 to 10 L/min.

A It is reassuring for the heart rate to elevate 15 bpm for at least 15 seconds with fetal scalp stimulation. The nurse should record the finding. No other intervention is necessary at this time.

After several mild late decelerations, the physician obtains a fetal scalp blood sample. The fetal pH was 7.32. The nurse is aware that the next action will probably be to: a. Continue to monitor the fetus during the labor. b. Prepare for a cesarean section. c. Prepare for a reassessment of the fetal pH. d. Apply oxygen to the mother at a rate of 10 L/min.

A The normal fetal pH is 7.25 to 7.35. Because this fetal pH is within the normal limits, the nurse can anticipate continuing monitoring of the fetus. Because the pH is within normal limits, it is unlikely that a cesarean section would be performed at this time.

The client in labor experiences a spontaneous rupture of membranes. What information related to this event must the nurse include in the client's record? a. Fetal heart rate b. Pain level c. Test results ensuring that the fluid is not urine d. The client's understanding of the event

ANS: A Charting related to membrane rupture includes the time, FHR, and character and amount of the fluid. Pain is not associated with this event. When it is obvious that the fluid is amniotic fluid, which is anticipated during labor, it is not necessary to verify this by testing. The client's understanding of the event would only need to be documented if it presents a problem.

Decelerations that mirror the contractions are present with each contraction on the monitor strip of a multipara who received epidural anesthesia 20 minutes ago. The nurse should: a. maintain the normal assessment routine. b. administer O2 at 8 to 10 L/min by face mask. c. increase the IV flow rate from 125 to 150 mL/hr. d. assess the maternal blood pressure for a systolic pressure below 100 mm Hg.

ANS: A Decelerations that mirror the contraction are early decelerations caused by fetal head compression. Early decelerations are not associated with fetal compromise and require no intervention. Administering O2, increasing the IV flow rate, and assessing for hypotension are not necessary in early decelerations.

The nurse is instructing a nursing student on the application of fetal monitoring devices. Which method of assessing the fetal heart rate requires the use of a gel? a. Doppler b. Fetoscope c. Scalp electrode d. Tocodynamometer

ANS: A Doppler is the only listed method involving ultrasonic transmission of fetal heart rates; it requires the use of a gel. The fetoscope does not require gel because ultrasonic transmission is not used. The scalp electrode is attached to the fetal scalp; gel is not necessary. The tocodynamometer does not require gel. This device monitors uterine contractions.

The most important nursing intervention for the patient who has received an epidural narcotic is a. Monitoring respiratory rate hourly b. Administering analgesics as needed c. Monitoring blood pressure every 4 hours d. Assessing the level of anesthesia

ANS: A Feedback A - The possibility of respiratory depression exists for up to 24 hours after administration of an epidural narcotic. B - Epidural narcotic should be enough pain relief that further medication is not necessary. Administering any other narcotic may cause an overdose. C - The patient's blood pressure needs to be monitored. However, that is not the major concern with this medication. D - The epidural narcotic should provide pain relief, but not anesthesia.

What is the first type of breathing technique used in labor? a. Slow-paced b. Modified-paced c. Patterned-paced d. Pant-blow

ANS: A Feedback A-Breathing for the first stage of labor consists of a cleansing breath and various breathing techniques known as paced breathing. The first type used in labor is the slow-paced. B-Modified-paced breathing is used when the slow-paced breathing is no longer effective. C-Patterned-paced breathing is used later in the labor and has the woman focusing on a pattern of breathing. D-Pant-blow breathing can be used to prevent pushing before the cervix is completely dilated.

To provide optimal care to the intrapartum woman, the nurse understands that the least favorable maternal position for labor is a. Supine b. Sitting c. Lying on the side d. Standing

ANS: A Feedback A-The supine position allows the heavy uterus to compress the inferior vena cava and can reduce placental blood flow, compromising fetal oxygen supply. B-The sitting position allows gravity to assist in the descent of the fetus. C-The side-lying position is comfortable for most mothers, and it allows for adequate placental blood flow. D-Standing and walking may be comfortable for some women. This does not prevent adequate placental blood flow and can be encouraged if there are no other contraindications.

Increasing the infusion rate of nonadditive intravenous fluids can increase fetal oxygenation primarily by: a. expanding the maternal blood volume. b. maintaining a normal maternal temperature. c. preventing normal maternal hypoglycemia. d. increasing the oxygen-carrying capacity of the maternal blood.

ANS: A Filling the mother's vascular system makes more blood available to perfuse the placenta and may correct hypotension. Increasing fluid volume may alter the maternal temperature only if she is dehydrated. Most IV fluids for laboring women are isotonic and do not add extra glucose. Oxygen-carrying capacity is increased by adding more red blood cells.

Which comfort measure should a nurse use to assist a laboring woman to relax? a. Recommend frequent position changes. b. Palpate her filling bladder every 15 minutes. c. Offer warm wet cloths to use on the client's face and neck. d. Keep the room lights lit so the client and her coach can see everything.

ANS: A Frequent maternal position changes reduce the discomfort from constant pressure and promote fetal descent. A full bladder intensifies labor pain. The bladder should be emptied every 2 hours. Women in labor get hot and perspire. Cool cloths are much better. Soft indirect lighting is more soothing than irritating bright lights.

Which is an essential part of nursing care for a laboring client? a. Helping the woman manage the pain b. Eliminating the pain associated with labor c. Feeling comfortable with the predictable nature of intrapartal care d. Sharing personal experiences regarding labor and birth to decrease her anxiety

ANS: A Helping a client manage the pain is an essential part of nursing care because pain is an expected part of normal labor and cannot be fully relieved. Labor pain cannot be fully relieved. The labor nurse should always be assessing for unpredictable occurrences. Decreasing anxiety is important, but managing pain is a top priority.

The nurse is preparing supplies for an amnioinfusion on a client with intact membranes. Which supplies should the nurse gather? (Select all that apply.) a. Extra underpads b. Solution of 3% normal saline c. Amniotic hook to perform an amniotomy d. Solid intrauterine pressure catheter with a pressure transducer on its tip

ANS: A, C Amnioinfusion is performed with lactated Ringer's solution or normal saline, not 3%. Normal saline is infused into the uterus through an intrauterine pressure catheter (IUPC). The underpads must be changed regularly because fluid leaks out constantly. The membranes need to be ruptured before an amnioinfusion can be initiated so an amniotic hook will be needed. The IUPC must have a double lumen to run the infusion through.

The nurse admits a laboring patient at term. On review of the prenatal record, the patient's pregnancy has been unremarkable and she is considered low risk. In planning the patient's care, at what interval will the nurse intermittently auscultate (IA) the fetal heart rate during the first stage of labor? a. Every 10 minutes b. Every 15 minutes c. Every 30 minutes d. Every 60 minutes

ANS: C Evaluate the fetal monitoring strip systematically for the elements noted. The following are recommended assessment and documentation intervals for IA and EFM (although facility policies may be different): low-risk women, every 30 minutes during the active phase and every 15 minutes during the second stage.

The nurse is caring for a client in the fourth stage of labor. Which assessment findings should the nurse identify as a potential complication? (Select all that apply.) a. Soft boggy uterus b. Maternal temperature of 99° F c. High uterine fundus displaced to the right d. Intense vaginal pain unrelieved by analgesics e. Half of a lochia pad saturated in the first hour after birth

ANS: A, C, D Assessment findings that may indicate a potential complication in the fourth stage include a soft boggy uterus, high uterine fundus displaced to the right, and intense vaginal pain unrelieved by analgesics. The maternal temperature may be slightly elevated after birth because of the inflammation to tissues, and half of a lochia pad saturated in the first hour after birth is within expected amounts.

The nurse is monitoring a client in the active stage of labor. Which conditions associated with fetal compromise should the nurse monitor? (Select all that apply.) a. Maternal hypotension b. Fetal heart rate of 140 to 150 bpm c. Meconium-stained amniotic fluid d. Maternal fever—38° C (100.4° F) or higher e. Complete uterine relaxation of more than 30 seconds between contractions

ANS: A, C, D Conditions associated with fetal compromise include maternal hypotension (may divert blood flow away from the placenta to ensure adequate perfusion of the maternal brain and heart), meconium-stained (greenish) amniotic fluid, and maternal fever (38° C [100.4° F] or higher). Fetal heart rate of 110 to 160 bpm for a term fetus is normal. Complete uterine relaxation is a normal finding.

A woman who is gravida 3 para 2 arrives on the intrapartum unit. What is the most important nursing assessment at this time? a.Contraction pattern, amount of discomfort, and pregnancy history b.FHR, maternal vital signs, and the woman's nearness to birth c.Identification of ruptured membranes, woman's gravida and para, and her support person d.Last food intake, when labor began, and cultural practices the couple desires

ANS: B All options describe relevant intrapartum nursing assessments; however, this focused assessment has a priority. If the maternal and fetal conditions are normal and birth is not imminent, then other assessments can be performed in an unhurried manner; these include: gravida, para, support person, pregnancy history, pain assessment, last food intake, and cultural practices.

Under which circumstance would it be unnecessary for the nurse to perform a vaginal examination? a.Admission to the hospital at the start of labor b.When accelerations of the FHR are noted c.On maternal perception of perineal pressure or the urge to bear down d.When membranes rupture

ANS: B An accelerated FHR is a positive sign; therefore, a vaginal examination would not be necessary. A vaginal examination should be performed when the woman is admitted to the hospital, when she perceives perineal pressure or the urge to bear down, when her membranes rupture, when a significant change in her uterine activity has occurred, or when variable decelerations of the FHR are noted.

29. The primipara at 39 weeks' gestation states to the nurse, "I can breathe easier now." What is the nurse's best response? a. "You labor will start any day now since the baby has dropped." b. "That process is called lightening. Do you have to urinate more frequently?" c. "Contact your health care provider when your contractions are every 5 minutes for 1 hour." d. "You will likely not feel you baby's movements as much now, so do not be concerned."

ANS: B As the fetus descends toward the pelvic inlet (dropping), the woman notices that she breathes more easily because upward pressure on her diaphragm is reduced. However, increased pressure on her bladder causes her to urinate more frequently. Pressure of the fetal head in the pelvis also may cause leg cramps and edema. Lightening (descent of the fetus toward the pelvic inlet before labor) is most noticeable in primiparas and occurs about 2 to 3 weeks before the natural onset of labor. Instructions for labor, although correct, do not address the patient's statement of being able to breathe easier. Fetal movement continues throughout the final weeks of gestation. A decrease in fetal movement is a concerning sign and the health care provider must be notified. PTS: 1 DIF: Cognitive Level: Application REF: 207

To clarify the fetal condition when baseline variability is absent, the nurse should first: a. monitor fetal oxygen saturation using fetal pulse oximetry. b. notify the physician so that a fetal scalp blood sample can be obtained. c. apply pressure to the fetal scalp with a glove finger using a circular motion. d. increase the rate of nonadditive IV fluid to expand the mother's blood volume.

ANS: C Fetal scalp stimulation helps identify whether the fetus responds to gentle massage. An acceleration in response to the massage suggests that the fetus is in normal oxygen and acid-base balance. Monitoring fetal oxygen saturation using fetal pulse oximetry is no longer available in the United States. Obtaining a fetal scalp blood sample is invasive and the results are not immediately available. Increasing the rate of nonadditive IV fluid would not clarify the fetal condition.

The nurse assesses the amniotic fluid. Which characteristic presents the lowest risk of fetal complications? a. Bloody b. Clear with bits of vernix caseosa c. Green and thick d. Yellow and cloudy with foul odor

ANS: B Amniotic fluid should be clear and may include bits of vernix caseosa, the creamy white fetal skin lubricant. Green fluid indicates that the fetus passed meconium before birth. The newborn may need extra respiratory suctioning at birth if the fluid is heavily stained with meconium. Cloudy, yellowish, strong-smelling, or foul-smelling fluid suggests infection. Bloody fluid may indicate partial placental separation.

A gravida 1, para 0, 38 weeks' gestation is in the transition phase of labor with SROM and is very anxious. Vaginal exam, 8 cm, 100% effaced, -1 station vertex presentation. She wants the nurse to keep checking her by performing repeated vaginal exams because she is sure that she is progressing rapidly. What is the best response that the nurse can provide to this client at this time? a. Performing more frequent vaginal exams will not make the labor go any quicker. b. Even though she is in transition, frequent vaginal exams must be limited because of the potential for infection. c. Tell the client that she will check every 30 minutes. d. Medicate the client as needed for anxiety so that the labor can progress.

ANS: B Data reveals a primipara in labor who is in transition (8 to 10 cm) with ruptured membranes. At this point, vaginal exams should be limited until the client feels further pressure and/or has increased bloody show, indicating fetal descent. Telling the client that performing more frequent vaginal exams will not make the labor go any quicker would not be therapeutic because this does not address client's anxiety. Telling the client that the nurse will continue checking every 30 minutes without adequate clinical indication is not the standard of care. Medicating the client is not an appropriate intervention at this time because effective communication will help alleviate stress, and the use of medications during transition may affect maternal and/or fetal well-being during birth.

When instructing the woman in early labor, the nurse teaches her that an important aspect of proper breathing technique is a. Breathing no more than three times the normal rate b. Beginning and ending with a cleansing breath c. Holding the breath no longer than 10 seconds d. Adhering exactly to the techniques as they were taught

ANS: B Feedback A-It is important to prevent hyperventilation; however, the cleansing breaths are the most important aspect of the breathing techniques. B-The cleansing breath helps the woman clear her mind to focus on relaxing and signals the coach that the contraction is beginning or ending. C-The woman should hold her breath for no more than 6 to 8 seconds. D-The woman needs to be flexible and change her breathing techniques as needed to keep her comfortable.

Which intervention may be most helpful for the patient experiencing most of her labor pain in her back? a. Lying supine with head slightly elevated b. Having the support person apply firm pressure to the sacrum c. Sitting upright with the legs straight d. Having the nurse massage the upper back during a contraction

ANS: B Feedback A-This would put more pressure onto the lower back area and decrease placental profusion. B-Firm pressure against the sacrum may be helpful in relieving the discomfort associated with back labor. C-This position would put more pressure onto the lower back area. D-The massage should be in the lower back where the pain is located.

The nurse is caring for a low-risk client in the active phase of labor. At which interval should the nurse assess the fetal heart rate? a. Every 15 minutes b. Every 30 minutes c. Every 45 minutes d. Every 1 hour

ANS: B For the fetus at low risk for complications, guidelines for frequency of assessments are at least every 30 minutes during the active phase of labor. 15-minute assessments would be appropriate for a fetus at high risk. 45-minute assessments during the active phase of labor are not frequent enough to monitor for complications. 1-hour assessments during the active phase of labor are not frequent enough to monitor for complications.

Which is the most appropriate method of intrapartum fetal monitoring when a woman has a history of hypertension during pregnancy? a. Continuous auscultation with a fetoscope b.Continuous electronic fetal monitoring c. Intermittent assessment with a Doppler transducer d. Intermittent electronic fetal monitoring for 15 minutes each hour

ANS: B Maternal hypertension may reduce placental blood flow through vasospasm of the spiral arteries. Reduced placental perfusion is best assessed with continuous electronic fetal monitoring to identify patterns associated with this condition. It is not practical to provide continuous auscultation with a fetoscope. This fetus needs continuous monitoring because it is at high risk for complications.

Under which circumstance should the nurse assist the laboring woman into a hands-and-knees position? a.Occiput of the fetus is in a posterior position. b.Fetus is at or above the ischial spines. c.Fetus is in a vertex presentation. d.Membranes have ruptured.

ANS: A The hands-and-knees position is effective in helping to rotate the fetus from a posterior to an anterior position. Many women experience the irresistible urge to push when the fetus is at the level of the ischial spines. In some cases, this urge is felt before the woman is fully dilated. The woman should be instructed not to push until complete cervical dilation has occurred. No one position is correct for childbirth. The two most common positions assumed by women are the sitting and side-lying positions. The woman may be encouraged into a hands-and-knees position if the umbilical cord prolapsed when the membranes ruptured.

The best time to teach nonpharmacologic pain control methods to an unprepared laboring client is during which stage? a. Latent phase b. Active phase c. Second stage d. Transition phase

ANS: A The latent phase of labor is the best time for intrapartum teaching because the woman is usually anxious enough to be attentive yet comfortable enough to understand the teaching. During the active phase, the woman is focused internally and unable to concentrate on teaching. During the second stage, the woman is focused on pushing. She normally handles the pain better at this point because she is active in doing something to hasten the birth. During transition, the woman is focused on keeping control; she is unable to focus on anyone else or learn at this time.

An 18-year-old primigravida is 4 cm dilated and her contractions are 5 minutes apart. She received little prenatal care. She had no childbirth preparation. She is crying loudly and shouting, "Please give me something for the pain. I can't take the pain." What is the priority nursing diagnosis? a. Pain related to uterine contractions b. Knowledge deficit related to the birth experience c. Ineffective coping related to inadequate preparation for labor d. Risk for injury related to lack of prenatal care

ANS: A The most important issue for this woman, at this time, is effective pain management.

A woman requests a pudendal block to manage her labor pain. The nurse realizes that the woman needs further explanation about the pudendal block when she says: a. "I'm having a contraction. Can I get the pudendal block now?" b. "I'll get the pudendal block right before I deliver." c. "The nurse midwife will insert the needles into my vagina." d. "It takes a few minutes after the medicine is administered to make me feel numb."

ANS: A The pudendal block does not block pain from contractions and is given just before birth.

13. A client just delivered a baby by the vaginal route. The client asks the nurse why the baby's head is not round, but oval. Which explanation should the nurse give to the client? a. This results from molding. b. This results from lightening. c. This results from the fetal lie. d. This results from the fetal presentation.

ANS: A The sutures and fontanels allow the bones of the fetal head to move slightly, changing the shape of the fetal head so it can adapt to the size and shape of the pelvis. Lightening is the descent of the fetus toward the pelvic inlet before labor. Lie is the relationship of the long axis of the fetus to the long axis of the mother. Presentation is the fetal part that first enters the pelvic outlet. PTS: 1 DIF: Cognitive Level: Application REF: 201

When assessing a woman in the first stage of labor, which clinical finding will alert the nurse that uterine contractions are effective? a.Dilation of the cervix b.Descent of the fetus to -2 station c.Rupture of the amniotic membranes d.Increase in bloody show

ANS: A The vaginal examination reveals whether the woman is in true labor. Cervical change, especially dilation, in the presence of adequate labor, indicates that the woman is in true labor. Engagement and descent of the fetus are not synonymous and may occur before labor. ROM may occur with or without the presence of labor. Bloody show may indicate a slow, progressive cervical change (e.g., effacement) in both true and false labor.

A laboring woman is reclining in the supine position. What is the most appropriate nursing action at this time? a.Ask her to turn to one side. b.Elevate her feet and legs. c.Take her blood pressure. d.Determine whether fetal tachycardia is present.

ANS: A The woman's supine position may cause the heavy uterus to compress her inferior vena cava, thus reducing blood return to her heart and reducing placental blood flow. Elevating her legs will not relieve the pressure from the inferior vena cava. If the woman is allowed to stay in the supine position and blood flow to the placental is reduced significantly, then fetal tachycardia may occur. The most appropriate nursing action is to prevent this from occurring by turning the woman to her side. Blood pressure readings may be obtained when the client is in the appropriate and safest position.

The nurse is caring for a client in early labor. Membranes ruptured approximately 2 hours earlier. This client is at increased risk for which complication? a.Intrauterine infection b.Hemorrhage c.Precipitous labor d.Supine hypotension

ANS: A When the membranes rupture, microorganisms from the vagina can ascend into the amniotic sac, causing chorioamnionitis and placentitis. ROM is not associated with fetal or maternal bleeding. Although ROM may increase the intensity of the contractions and facilitate active labor, it does not result in precipitous labor. ROM has no correlation with supine hypotension.

At 1 minute after birth, the nurse assesses the newborn to assign an Apgar score. The apical heart rate is 110 bpm, and the infant is crying vigorously with the limbs flexed. The infant's trunk is pink, but the hands and feet are blue. The Apgar score for this infant is: a. 7. b. 8. c. 9. d. 10.

ANS: C The Apgar score is 9 because 1 point is deducted from the total score of 10 for the infant's blue hands and feet. The baby received 2 points for each of the categories except color. Because the infant's hands and feet were blue, this category is given a grade of 1. The baby received 2 points for each of the categories except color. Because the infant's hands and feet were blue, this category is given a grade of 1. The infant had 1 point deducted because of the blue color of the hands and feet.

The nurse assists the midwife during a vaginal examination of the client in labor. What does the nurse recognize as the primary reason that a vaginal exam is done at this time? a. To apply internal monitoring electrodes b. To assess for Goodell's sign c. To determine cervical dilation and effacement d. To determine strength of contractions

ANS: C The primary purpose of a vaginal exam during labor is to determine cervical dilation and effacement and fetal descent. Goodell's sign is assessed in early pregnancy, not during labor. Although application of monitoring electrodes is done by entering the vagina, it is not the primary purpose of a vaginal exam. Vaginal exams are not done to determine the strength of contractions.

Proper placement of the tocotransducer for electronic fetal monitoring is: a. inside the uterus. b. on the fetal scalp. c. over the uterine fundus. d. over the mother's lower abdomen.

ANS: C The tocotransducer monitors uterine activity and should be placed over the fundus, where the most intensive uterine contractions occur. The tocotransducer is for external use. The tocotransducer monitors uterine contractions. The most intensive uterine contractions occur at the fundus; this is the best placement area.

6. The laboring client asks the nurse how the labor contractions work to dilate the cervix. The best response by the nurse is that labor contractions facilitate cervical dilation by: a. promoting blood flow to the cervix. b. contracting the lower uterine segment. c. enlarging the internal size of the uterus. d. pulling the cervix over the fetus and amniotic sac.

ANS: D Effective uterine contractions pull the cervix upward at the same time the fetus and amniotic sac are pushed downward. Blood flow decreases to the uterus during a contraction. The contractions are stronger at the fundus. The internal size becomes smaller with the contractions; this helps push the fetus down. PTS: 1 DIF: Cognitive Level: Application REF: 198

Which method of pain management would be safest for a gravida 3, para 2, admitted at 8 cm cervical dilation? a. Narcotics b. Spinal block c. Epidural anesthesia d. Breathing and relaxation techniques

ANS: D Nonpharmacologic methods of pain management may be the best option for a woman in advanced labor. At 8 cm cervical dilation there probably not enough time remaining to administer spinal anesthesia or epidural anesthesia. A narcotic given at this time may reach its peak at about the time of birth and result in respiratory depression in the newborn.

A patient in active labor requests an epidural for pain management. What is the nurse's priority action for this patient? a. Assess the fetal heart rate pattern over the next 30 minutes. b. Take the patient's blood pressure every 5 minutes for 15 minutes. c. Determine the patient's contraction pattern for the next 30 minutes. d. Initiate an IV infusion of lactated Ringer's solution at 2000 mL/hr over 30 minutes.

ANS: D Rapid infusion of a nondextrose IV solution, often warmed, such as lactated Ringer's or normal saline, before initiation of the block fills the vascular system to offset vasodilation. Preload IV quantities are at least 500 to 1000 mL infused rapidly. Vasodilation with corresponding hypotension can reduce placental perfusion and is most likely to occur within the first 15 minutes after the initiation of the epidural. Determining the fetal heart rate every 30 minutes is the standard of care. The patient is in active labor, which indicates a contraction pattern resulting in cervical dilation.

Which collection of risk factors will most likely result in damaging lacerations, including episiotomies? a.Dark-skinned woman who has had more than one pregnancy, who is going through prolonged second-stage labor, and who is attended by a midwife b.Reddish-haired mother of two who is going through a breech birth c.Dark-skinned first-time mother who is going through a long labor d.First-time mother with reddish hair whose rapid labor was overseen by an obstetrician

ANS: D Reddish-haired women have tissue that is less distensible than darker-skinned women and therefore may have less efficient healing. First-time mothers are also at greater risk, especially with breech births, long second-stage labors, or rapid labors during which the time for the perineum to stretch is insufficient. The rate of episiotomies is higher when obstetricians rather than midwives attend the births. The woman in the first scenario (a) is at low risk for either damaging lacerations or an episiotomy. She is multiparous, has dark skin, and is being attended by a midwife, who is less likely to perform an episiotomy. Reddish-haired women have tissue that is less distensible than that of darker-skinned women. Consequently, the client in the second scenario (b) is at increased risk for lacerations; however, she has had two previous deliveries, which result in a lower likelihood of an episiotomy. The fact that the woman in the third scenario (c) is experiencing a prolonged labor might increase her risk for lacerations. Fortunately, she is dark skinned, which indicates that her tissue is more distensible than that of fair-skinned women and therefore less susceptible to injury.

12. A nullipara client has progressed to the active phase of labor. The nurse understands that this phase of labor, on the average, for a nullipara will last how long? a. 50 minutes b. hours c. 6 to 7 hours d. 8 to 10 hours

ANS: D The active phase of labor for a nullipara lasts 8 to 10 hours. The second phase of labor lasts 50 minutes for a nullipara. The transition phase lasts hours for a nullipara. A multipara's active phase of labor is 6 to 7 hours. PTS: 1 DIF: Cognitive Level: Analysis REF: 208

The Valsalva maneuver can be described as the process of making a forceful bearing-down attempt while holding one's breath with a closed glottis and a tightening of the abdominal muscles. When is it appropriate to instruct the client to use this maneuver? a.During the second stage to enhance the movement of the fetus bDuring the third stage to help expel the placenta c.During the fourth stage to expel blood clots d.Not at all

ANS: D The client should not be instructed to use this maneuver. This process stimulates the parasympathetic division of the autonomic nervous system and produces a vagal response (decrease in heart rate and blood pressure.) An alternative method includes instructing the client to perform open-mouth and open-glottis breathing and pushing.

The nurse performs a vaginal examination to assess a client's labor progress. Which action should the nurse take next? a.Perform an examination at least once every hour during the active phase of labor. b.Perform the examination with the woman in the supine position. c.Wear two clean gloves for each examination. d.Discuss the findings with the woman and her partner

ANS: D The nurse should discuss the findings of the vaginal examination with the woman and her partner, as well as report the findings to the primary care provider. A vaginal examination should be performed only when indicated by the status of the woman and her fetus. The woman should be positioned so as to avoid supine hypotension. The examiner should wear a sterile glove while performing a vaginal examination for a laboring woman.

17. A client in labor presents with a breech presentation. The nurse understands that a breech presentation is associated with: a. more rapid labor. b. a high risk of infection. c. maternal perineal trauma. d. umbilical cord compression.

ANS: D The umbilical cord can compress between the fetal body and maternal pelvis when the body has been born but the head remains within the pelvis. Breech presentation is not associated with a more rapid labor. There is no higher risk of infection with a breech birth. There is no higher risk for perineal trauma with a breech birth. PTS: 1 DIF: Cognitive Level: Understanding REF: 202, 203

The nurse has given the newborn an Apgar score of 5. She should then: a. begin ventilation and compressions. b. do nothing except place the infant under a radiant warmer. c. observe the infant and recheck the score after 10 minutes. d. gently stimulate by rubbing the infant's back while administering O2.

ANS: D An infant who receives a score of 4 to 6 requires only additional oxygen and gentle stimulation. An infant who receive a score of 3 or less requires ventilation and compressions. An infant who scores less than 7 requires more intervention than placement under a radiant warmer. Observing and rechecking the infant will not improve newborn's transition to extrauterine life.

The nurse is concerned that a client's uterine activity is too intense and that her obesity is preventing accurate assessment of the actual intrauterine pressure. Based on this information, which action should the nurse take? a. Reposition the tocotransducer. b. Reposition the Doppler transducer. c. Obtain an order from the health care provider for a spiral electrode. d. Obtain an order from the health care provider for an intrauterine pressure catheter.

ANS: D An intrauterine pressure catheter can measure actual intrauterine pressure. The tocotransducer measures the uterine pressure externally; this would not be accurate with an obese client, even with repositioning. A Doppler auscultates the FHR. A scalp electrode (or spiral electrode) measures the fetal heart rate (FHR).

One of the greatest risks to the mother during administration of general anesthesia is a. Respiratory depression b. Uterine relaxation c. Inadequate muscle relaxation d. Aspiration of stomach contents

ANS: D Feedback A-Respirations can be altered during general anesthesia, and the anesthesiologist will take precautions to maintain proper oxygenation. B-Uterine relaxation can occur with some anesthesia, but this can be monitored and prevented. C-Inadequate muscle relaxation can be altered. This is not the greatest risk for the mother. D -Aspiration of acidic gastric contents and possible airway obstruction is a potentially fatal complication of general anesthesia.

Which of the following therapeutic applications provides the most accurate information related to uterine contraction strength? a. External fetal monitoring (EFM) b. Internal fetal monitoring c. Intrauterine pressure catheter (IUPC) d. Maternal comments based on perception

ANS: C IUPC is a clinical tool that provides an accurate assessment of uterine contraction strength. EFM provides evidence of contraction pattern and fetal heart rate but only estimates uterine contraction strength. Internal fetal monitoring provides direct evidence of fetal heart rate and contraction pattern. It only estimates uterine contraction strength. Maternal comments related to pain may not be related to uterine contraction strength and thus are influenced by the client's own pain perception.

If the position of a fetus in a cephalic presentation is right occiput anterior, the nurse should assess the fetal heart rate in which quadrant of the maternal abdomen? a. Right upper b. Left upper c. Right lower d. Left lower

ANS: C If the fetus is in a right occiput anterior position, the fetal spine will be on the mother's right side. The best location to hear the fetal heart rate is through the fetal shoulder, which would be in the right lower quadrant. The right upper, left upper, and left lower areas are not the best locations for assessing the fetal heart rate in this case.

Which technique could the support person use when the laboring woman appears to be losing control? a. Have the nurse take over the role of support. b. Tell the woman that she is causing stress to her baby and herself. c. Wait for the contraction to end and discuss the problem with her. d. Make eye contact with the woman and breathe along with her.

ANS: D Feedback A-The woman already has a trusting relationship with the support person so they should stay in that position if possible. B-Imagery involves the woman creating a relaxing mental scene and dissociating herself from the painful aspects of labor. C-Mental stimulation occupies the woman's mind and competes with pain stimuli. D-Making eye contact and breathing along with the laboring woman to help pace her breathing will assist her in remaining calm.

Which type of cutaneous stimulation involves massage of the abdomen? a. Thermal stimulation b. Imagery c. Mental stimulation d. Effleurage

ANS: D Feedback A-Thermal stimulation is the use of warmth to provide comfort, such as showers and baths. B-Imagery involves the woman creating a relaxing mental scene and dissociating herself from the painful aspects of labor. C- Mental stimulation occupies the woman's mind and competes with pain stimuli. D-Effleurage is massage usually performed on the abdomen during contractions.

Late deceleration patterns are noted when assessing the monitor tracing of a woman whose labor is being induced with an infusion of oxytocin (Pitocin). The woman is in a side-lying position and her vital signs are stable, falling within a normal range. Contractions are intense, last 90 seconds, and occur every 1½ to 2 minutes. The nurse's immediate action would be to: a. Change the woman's position. b. Stop the oxytocin. c. Elevate the woman's legs. d. Administer oxygen via a tight mask at 8 to 10 L/min.

B The late deceleration patterns noted are most likely related to alteration in uteroplacental perfusion associated with the strong contractions described. The immediate action would be to stop the oxytocin infusion because oxytocin stimulates the uterus to contract. The woman is already in an appropriate position for uteroplacental perfusion. Elevating her legs would be appropriate if hypotension were present. Oxygen is appropriate but not the immediate action.

A 25-year-old primigravida client is in the first stage of labor. She and her husband have been holding hands and breathing together through each contraction. Suddenly, the client pushes her husband's hand away and shouts, "Don't touch me!" This behavior is most likely: a. abnormal labor. b. a sign that she needs analgesia. c. normal and related to hyperventilation. d. common during the transition phase of labor.

ANS: D The transition phase of labor is often associated with an abrupt change in behavior, including increased anxiety and irritability. This change of behavior is an expected occurrence during the transition phase. If she is in the transitional phase of labor, analgesia may not be appropriate if the birth is near. Hyperventilation will produce signs of respiratory alkalosis.

Why is continuous electronic fetal monitoring generally used when oxytocin is administered? a. Fetal chemoreceptors are stimulated. b. The mother may become hypotensive. c. Maternal fluid volume deficit may occur. d. Uteroplacental exchange may be compromised.

ANS: D The uterus may contract more firmly and the resting tone may be increased with oxytocin use. This response reduces the entrance of freshly oxygenated maternal blood into the intervillous spaces, depleting fetal oxygen reserves. Oxytocin affects the uterine muscles. Hypotension is not a common side effect of oxytocin. All laboring women are at risk for fluid volume deficit; oxytocin administration does not increase the risk.

When a pattern of variable decelerations occur, the nurse should: a. administer O2 at 8 to 10 L/min. b. place a wedge under the right hip. c. increase the IV fluids to 150 mL/hr. d. position client in a knee-chest position.

ANS: D Variable decelerations are caused by conditions that reduce flow through the umbilical cord. The client should be repositioned when the FHR pattern is associated with cord compression. The knee-chest position uses gravity to shift the fetus out of the pelvis to relieve cord compression. Administering oxygen will not be effective until cord compression is relieved. Increasing the IV fluids and placing a wedge under the right hip are not effective interventions for cord compression.

A 39-week primigravida calls the birthing center and tells the nurse she has contractions that are 10 to 15 minutes apart and had a small gush of fluid about 1 hour ago. The nurse should tell her to: a. Wait until the contractions are about 5 minutes apart and come to the center. b. Come to the birthing center now. c. Come to the birthing center in about an hour if she lives farther than 1 hour away. d. Come to the birthing center if the baby stops moving.

B A gush or trickle of fluid from the vagina should be evaluated as soon as possible. Waiting until the contractions are 5 minutes apart is appropriate for a primigravida if the membranes have not ruptured.

In caring for a low-risk woman in the active phase of labor, the nurse realizes the assessment of fetal well-being should occur: a. Every 15 minutes. b. Every 30 minutes. c. Every 5 minutes. d. Every hour.

B For low-risk women, the nurse should evaluate the fetal monitoring strip or assessment fetal well-being at least every 30 minutes during the active phase of labor and every 15 minutes during the second stage. For the high-risk woman, monitoring should occur every 15 minutes during the active phase and every 5 minutes during the second stage.

The nurse is preparing to auscultate the fetal heart rate using a Doppler transducer. When performing the Leopold maneuver, the nurse felt the buttocks near the fundus and the back along the left side of the mother. The best position for the Doppler would be in the mother's: a. Left upper quadrant. b. Left lower quadrant. c. Right upper quadrant. d. Right lower quadrant.

B The fetal heart is best heard through the fetus's upper back. Because this fetus is in a cephalic position, with the back toward the mother's left side, the Doppler should be placed in the left lower quadrant of the mother's abdomen.

After birth, the nurse assesses the newborn. The heart rate is 90 bpm, the body is flexed, there is vigorous movement, the newborn is actively crying when stimulated, and has bluish coloration in the feet and hands. The proper Apgar score for this newborn should be: a. 7. b. 8. c. 9. d. 10.

B The heart rate less than 100 bpm gets a score of 1, a lusty cry will give a score of 2 for both respiratory effort and reflex response, the flexed posture and vigorous movements gives a score of 2, and the bluish coloration of the hands and feet will give a score of 1.

Fluctuations in the baseline FHR that cause the printed line to have an irregular rather than a smooth appearance is termed ___________________.

Variability

During labor a vaginal examination should be performed only when necessary because of the risk of: a. infection. b. fetal injury. c. discomfort. d. perineal trauma.

ANS: A Vaginal examinations increase the risk of infection by carrying vaginal microorganisms upward toward the uterus. Properly performed vaginal examinations should not cause fetal injury. Vaginal examinations may be uncomfortable for some women in labor, but that is not the main reason for limiting them. A properly performed vaginal examination should not cause perineal trauma.

When giving a narcotic to a laboring client, which statement explains why the nurse should inject the medication at the beginning of a contraction? a. The medication will be rapidly circulated. b. Less medication will be transferred to the fetus. c. The maternal vital signs will not be adversely affected. d. Full benefit of the medication is received during that contraction.

ANS: B Injecting at the beginning of a contraction, when blood flow to the placenta is normally reduced, limits transfer to the fetus. It will not increase the circulation of the medication. It will not alter the vital signs any more than giving it at another time. The full benefit will be received by the woman, but it will decrease the amount reaching the fetus.

Which of the following are important points when teaching a client the proper method for pushing during the second stage of labor? (Select all that apply). a. Begin and end by taking a deep breath and exhaling. b. Push for 4 to 6 seconds at a time. c. Take a deep breath and then push while holding her breath. d. Push at least five or six times with each contraction.

A, B

Which of these might cause late decelerations in the fetus? (Select all that apply). a. Maternal hypotension b. Excessive uterine activity c. Maternal hypertension d. Fever e. Maternal overhydration f. Prolapsed cord

A, B, C

Labor pain management may include which of the following interventions? (Select all that apply). a. Cool, damp washcloths on the face and neck b. Decreasing bright lights in the room c. Keeping the woman clean and dry d. Administering pain medication as ordered f. Offering simple snacks every 2 hours

A, B, C, D

Childbirth preparation can be considered successful if the outcome is described as which of the following? a. Labor was pain-free. b. The birth experiences of friends and families were ignored. c. Only nonpharmacologic methods for pain control were used. d. The client rehearsed labor and practiced skills to master pain.

ANS: D Preparation allows the woman to rehearse for labor and to learn new skills to cope with the pain of labor and the expected behavioral changes. Childbirth preparation does not guarantee a pain-free labor. A woman should be prepared for pain and anesthesia-analgesia realistically. Friends and families can be an important source of support if they convey realistic information about labor pain. Women will not always achieve their desired level of pain control by using nonpharmacologic methods alone

4. Uncontrolled maternal hyperventilation during labor results in: a. metabolic acidosis. b. metabolic alkalosis. c. respiratory acidosis. d. respiratory alkalosis.

ANS: D Rapid deep respirations cause the laboring woman to lose carbon dioxide through exhalation, resulting in respiratory alkalosis. Hyperventilation does not cause respiratory acidosis, metabolic acidosis, or metabolic alkalosis. PTS: 1 DIF: Cognitive Level: Understanding REF: 198

Which component of the physical examination are Leopold's maneuvers unable to determine? a.Gender of the fetus b.Number of fetuses c.Fetal lie and attitude d.Degree of the presenting part's descent into the pelvis

ANS: A Leopold's maneuvers help identify the number of fetuses, the fetal lie and attitude, and the degree of descent of the presenting part into the pelvis. The gender of the fetus cannot be determined by performing Leopold's maneuvers.

19. Which maternal factor may inhibit fetal descent? a. A full bladder b. Decreased peristalsis c. Rupture of membranes d. Reduction in internal uterine size

ANS: A A full bladder may inhibit fetal descent because it occupies space in the pelvis needed by the fetal presenting part. Peristalsis does not influence fetal descent. Rupture of membranes will assist in the fetal descent. Contractions will reduce the internal uterine size to assist fetal descent. PTS: 1 DIF: Cognitive Level: Understanding REF: 198

Upon completion of a vaginal examination on a laboring woman, the nurse records: 50%, 6 cm, -1. Which of the following is a correct interpretation of the data? a. Fetal presenting part is 1 cm above the ischial spines. b. Effacement is 4 cm from completion . c. Dilation is 50% completed . d. Fetus has passed through the ischial spines.

A A station of -1 indicates that the fetal presenting part is above the ischial spines and has not yet passed through the pelvic inlet. A station of 0 would indicate that the presenting part has passed through the inlet and is at the level of the ischial spines or is engaged. Passage through the ischial spines with internal rotation would be indicated by a plus station, such as +1. Progress of effacement is referred to by percentages, with 100% indicating full effacement, and dilation by centimeters (cm), with 10 cm indicating full dilation.

The nurse in the birth room receives an order to give a newborn 0.3 mg of naloxone (Narcan) intramuscularly. The medication vial reads naloxone (Narcan), 0.4 mg/mL. The nurse should prepare how many milliliters to administer the correct dose? Fill in the blank and record your answer using two decimal places. _____ mL

0.75

On review of a fetal monitor tracing, the nurse notes that for several contractions the FHR decelerates as a contraction begins and returns to baseline just before it ends. The nurse should: a. Describe the finding in the notes. b. Reposition the woman onto her side. c. Call the physician for instructions. d. Administer oxygen at 8 to10 L/min with a tight face mask.

A An early deceleration pattern from head compression is described. No action other than documentation of the finding is required because this is an expected reaction to compression of the fetal head as it passes through the cervix. The other responses would be implemented when nonreassuring or ominous changes are noted.

The vaginal examination is an essential component of labor assessment. It reveals whether the client is in true labor and enables the examiner to determine whether membranes have ruptured. The vaginal examination is often stressful and uncomfortable for the client and should be performed only when indicated. Match the correct step number, from 1 to 7, with each component of a vaginal examination of the laboring woman. a.After obtaining permission, gently insert the index and middle fingers into the vagina. b.Explain the findings to the client. c.Position the woman to prevent supine hypotension. d.Use sterile gloves and soluble gel for lubrication. e.Document findings and report to the health care provider. f.Cleanse the perineum and vulva, if necessary. g.Determine dilation, presenting part, status of membranes, and characteristics of amniotic fluid. 1. Step 1 2. Step 2 3. Step 3 4. Step 4 5. Step 5 6. Step 6 7. Step 7

1. ANS: D DIF: Cognitive Level: Apply REF: p. 444 TOP: Nursing Process: Assessment MSC: Client Needs: Physiologic Integrity NOT: The vaginal examination should be performed on admission, before administering analgesics, when a significant change in uterine activity has occurred, on maternal perception of perineal pressure, when membranes rupture, or when the nurse notes variable decelerations of the FHR. A full explanation of the examination and support of the woman are important in reducing the level of stress and discomfort. 2. ANS: C DIF: Cognitive Level: Apply REF: p. 444 TOP: Nursing Process: Assessment MSC: Client Needs: Physiologic Integrity NOT: The vaginal examination should be performed on admission, before administering analgesics, when a significant change in uterine activity has occurred, on maternal perception of perineal pressure, when membranes rupture, or when the nurse notes variable decelerations of the FHR. A full explanation of the examination and support of the woman are important in reducing the level of stress and discomfort. 3. ANS: F DIF: Cognitive Level: Apply REF: p. 444 TOP: Nursing Process: Assessment MSC: Client Needs: Physiologic Integrity NOT: The vaginal examination should be performed on admission, before administering analgesics, when a significant change in uterine activity has occurred, on maternal perception of perineal pressure, when membranes rupture, or when the nurse notes variable decelerations of the FHR. A full explanation of the examination and support of the woman are important in reducing the level of stress and discomfort. 4. ANS: A DIF: Cognitive Level: Apply REF: p. 444 TOP: Nursing Process: Assessment MSC: Client Needs: Physiologic Integrity NOT: The vaginal examination should be performed on admission, before administering analgesics, when a significant change in uterine activity has occurred, on maternal perception of perineal pressure, when membranes rupture, or when the nurse notes variable decelerations of the FHR. A full explanation of the examination and support of the woman are important in reducing the level of stress and discomfort. 5. ANS: G DIF: Cognitive Level: Apply REF: p. 444 TOP: Nursing Process: Assessment MSC: Client Needs: Physiologic Integrity NOT: The vaginal examination should be performed on admission, before administering analgesics, when a significant change in uterine activity has occurred, on maternal perception of perineal pressure, when membranes rupture, or when the nurse notes variable decelerations of the FHR. A full explanation of the examination and support of the woman are important in reducing the level of stress and discomfort. 6. ANS: B DIF: Cognitive Level: Apply REF: p. 444 TOP: Nursing Process: Assessment MSC: Client Needs: Physiologic Integrity NOT: The vaginal examination should be performed on admission, before administering analgesics, when a significant change in uterine activity has occurred, on maternal perception of perineal pressure, when membranes rupture, or when the nurse notes variable decelerations of the FHR. A full explanation of the examination and support of the woman are important in reducing the level of stress and discomfort. 7. ANS: E DIF: Cognitive Level: Apply REF: p. 444 TOP: Nursing Process: Assessment MSC: Client Needs: Physiologic Integrity NOT: The vaginal examination should be performed on admission, before administering analgesics, when a significant change in uterine activity has occurred, on maternal perception of perineal pressure, when membranes rupture, or when the nurse notes variable decelerations of the FHR. A full explanation of the examination and support of the woman are important in reducing the level of stress and discomfort.

Match each term with the correct definition. a. The fetal part that enters the pelvic inlet first b. The orientation of the long axis of the fetus to the long axis of the woman c. Relation of a fixed reference point on the fetus to the quadrants of the maternal pelvis 1. Position 2. Fetal lie 3. Presentation

1.c 2.b 3.a The position is the relation of a fixed reference point on the fetus to the quadrants of the maternal pelvis. The fetal lie is the orientation of the long axis of the fetus to the long axis of the woman. The presentation is the fetal part that enters the pelvic inlet first.

A nurse documents that the fetal heart rate variability is marked. This indicates that the range is greater than how many beats per minute? Record your answer as a whole number. _____ bpm

25 There are four categories of fetal heart rate variability: Absent: Amplitude range is undetectable Minimal: detectable to less than or equal to 5 beats/min Moderate (normal): 6 to 25 beats/min Marked: Range >25 beats/min

Mathching: 42. Early decelerations 43. Late decelerations 44. Variable decelerations a. Caused by umbilical cord compression b. Caused by fetal head compression c. Caused by uteroplacental insufficiency

42. B - Fetal head compression briefly increases intracranial pressure, causing the vagus nerve to slow the heart rate. Deficient exchange of oxygen and waste products in the placenta (uteroplacental insufficiency) may result in a pattern of late (delayed) decelerations. Conditions that reduce flow through the umbilical cord may result in variable decelerations. 43. C - Fetal head compression briefly increases intracranial pressure, causing the vagus nerve to slow the heart rate. Deficient exchange of oxygen and waste products in the placenta (uteroplacental insufficiency) may result in a pattern of late (delayed) decelerations. Conditions that reduce flow through the umbilical cord may result in variable decelerations. 44. A - Fetal head compression briefly increases intracranial pressure, causing the vagus nerve to slow the heart rate. Deficient exchange of oxygen and waste products in the placenta (uteroplacental insufficiency) may result in a pattern of late (delayed) decelerations. Conditions that reduce flow through the umbilical cord may result in variable decelerations.

An intravenous access is started in most labor clients because of which of the following? (Select all that apply). a. To have quick access if drugs are needed b. To provide fluids to prevent dehydration c. In case an epidural block is administered d. To provide a route for pain medications for the 48-hour postpartum period

A, B, C By 48 hours postpartum, mothers are expected to be on oral pain medication.

Which of the following are considered nonreassuring fetal heart rate patterns? (Select all that apply). a. Tachycardia b. Bradycardia c. Absent variability d. Early decelerations e. Variable decelerations

A, B, C, E

During the labor process, the client's membranes rupture. Select all the assessments that are necessary for the nurse to carry out at this time. a. Color of amniotic fluid b. Odor of amniotic fluid c. Fetal heart rate d. Cervical dilatation e. Cervical effacement f. Time the membranes ruptured

A, B, C, F

Variability can be reduced by which of the following factors? (Select all that apply). a. Sleep b. Narcotics c. Gestation longer than 39 weeks d. Fetal anomalies that affect the central nervous system

A, B, D

To ensure adequate fetal oxygenation, which of the following are needed? (Select all that apply). a. Normal maternal blood flow and volume to the placenta Correct b. Normal oxygen saturation in maternal blood c. Normal carbon dioxide saturation in the maternal blood d. Adequate exchange of oxygen and carbon dioxide in the placenta e. Normal fetal circulatory and oxygen-carrying functions f. Normal blood glucose levels in the fetal circulation

A, B, D, E

A laboring client who imagines her body opening to let the baby out is using a mental technique called: a. imagery. b. effleurage. c. distraction. d. dissociation.

ANS: A Imagery is a technique of visualizing images that will assist the woman in coping with labor. Effleurage is self-massage. Distraction can be used in the early latent phase by having the woman involved in another activity. Dissociation helps the woman learn to relax all muscles except those that are working.

Which information regarding the procedures and criteria for admitting a woman to the hospital labor unit is important for the nurse to understand? a.Client is considered to be in active labor when she arrives at the facility with contractions. b.Client can have only her male partner or predesignated doula with her at assessment. c.Children are not allowed on the labor unit. d.Non-English speaking client must bring someone to translate.

ANS: A According to the Emergency Medical Treatment and Active Labor Act (EMTALA), a woman is entitled to active labor care and is presumed to be in true labor until a qualified health care provider certifies otherwise. A woman may have anyone she wishes present for her support. An interpreter must be provided by the hospital, either in person or by a telephonic service. Siblings of the new infant may be allowed at the delivery, depending on hospital policy and adequate preparation and supervision.

When a pregnant woman arrives at the labor suite, she tells the nurse that she wants to have an epidural for delivery. The nurse is aware that the factor that would be a contraindication to an epidural block is: a. A low platelet count b. Previous cesarean delivery c. A history of migraine headaches d. A history of diabetes mellitus

ANS: A An epidural block is not used if a woman has abnormal blood clotting.

Excessive anxiety during labor heightens the client's sensitivity to pain by increasing: a. muscle tension. b. the pain threshold. c. blood flow to the uterus. d. rest time between contractions.

ANS: A Anxiety and fear increase muscle tension, diverting oxygenated blood to the woman's brain and skeletal muscles. Prolonged tension results in general fatigue, increased pain perception, and reduced ability to use coping skills. Anxiety will decrease the pain threshold. Anxiety can decrease blood flow to the uterus. Anxiety will decrease the amount of rest the mother gets between contractions.

11. An increase in urinary frequency and leg cramps after the 36th week of pregnancy most likely indicates: a. lightening. b. breech presentation. c. urinary tract infection. d. onset of Braxton-Hicks contractions.

ANS: A As the fetus descends toward the pelvic inlet near the end of pregnancy, increased pelvic pressure occurs, resulting in greater urinary frequency and more leg cramps. Breech presentation does not cause urinary frequency and leg cramps. A urinary tract infection may cause urinary frequency but with burning and would not cause leg cramps. Braxton-Hicks contractions are irregular and mild and occur throughout the pregnancy. PTS: 1 DIF: Cognitive Level: Understanding REF: 207

21. Which clinical findings would be considered to be normal for a preterm fetus during the labor period? a. Baseline tachycardia b. Baseline bradycardia c. Fetal anemia d. Acidosis

ANS: A Because the nervous system is immature, it is expected that the preterm fetus will have a baseline tachycardia because of stimulation of the sympathetic nervous system. Baseline bradycardia, fetal anemia, and acidosis would indicate abnormal findings and fetal compromise. PTS: 1 DIF: Cognitive Level: Application REF: 199

When managing the care of a woman in the second stage of labor, the nurse uses various measures to enhance the progress of fetal descent. Which instruction best describes these measures? a.Encouraging the woman to try various upright positions, including squatting and standing b.Telling the woman to start pushing as soon as her cervix is fully dilated c.Continuing an epidural anesthetic so pain is reduced and the woman can relax d.Coaching the woman to use sustained, 10- to 15-second, closed-glottis bearing-down efforts with each contraction

ANS: A Both upright and squatting positions may enhance the progress of fetal descent. Many factors dictate when a woman should begin pushing. Complete cervical dilation is necessary, but complete dilation is only one factor. If the fetal head is still in a higher pelvic station, then the physician or midwife may allow the woman to "labor down" if the woman is able (allowing more time for fetal descent and thereby reducing the amount of pushing needed). The epidural may mask the sensations and muscle control needed for the woman to push effectively. Closed glottic breathing may trigger the Valsalva maneuver, which increases intrathoracic and cardiovascular pressures, reducing cardiac output and inhibiting perfusion of the uterus and placenta. In addition, holding her breath for longer than 5 to 7 seconds diminishes the perfusion of oxygen across the placenta and results in fetal hypoxia.

26. To determine if the client is in true labor, the nurse would assess for changes in: a. cervical dilation. b. amount of bloody show. c. fetal position and station. d. pattern of uterine contractions.

ANS: A Cervical changes are the only indication of true labor and are used to determine true and false labor. Changes in the amount of bloody show, fetal position and station, and pattern of uterine contractions are unreliable indicators of true labor. PTS: 1 DIF: Cognitive Level: Analysis REF: 208

As the United States and Canada continue to become more culturally diverse, recognizing a wide range of varying cultural beliefs and practices is increasingly important for the nursing staff. A client is from which country if she requests to have the baby's father in attendance? a.Mexico b.China c.Iran d.India

ANS: A Hispanic women routinely have fathers and female relatives in attendance during the second stage of labor. The father of the baby is expected to provide encouragement, support, and reassurance that all will be well. In China, fathers are usually not present. The side-lying position is preferred for labor and birth because it is believed that this will reduce trauma to the infant. In China, the client has a stoic response to pain. In Iran, the father will not be present. Female support persons and female health care providers are preferred. For many, a male caregiver is unacceptable. In India, the father is usually not present, but female relatives are usually in attendance. Natural childbirth methods are preferred

In recovery, if a woman is asked to either raise her legs (knees extended) off the bed or flex her knees, and then place her feet flat on the bed and raise her buttocks well off the bed, the purpose of this exercise is to assess what? a.Recovery from epidural or spinal anesthesia b.Hidden bleeding underneath her c.Flexibility d.Whether the woman is a candidate to go home after 6 hours

ANS: A If the numb or prickly sensations are gone from her legs after these movements, then she has likely recovered from the epidural or spinal anesthesia. Assessing the client for bleeding beneath her buttocks before discharge from the recovery is always important; however, she should be rolled to her side for this assessment. The nurse is not required to assess the woman for flexibility. This assessment is performed to evaluate whether the client has recovered from spinal anesthesia, not to determine if she is a candidate for early discharge.

32. After birth of the placenta the patient states, "All of a sudden I feel very cold." What is the best nursing action in response to this statement? a. Place a warm blanket over the patient. b. Place the baby on the patient's abdomen. c. Tell the patient that chills are expected after birth. d. "What do you mean by your words 'very cold'?"

ANS: A Many women are chilled after birth. The cause of this reaction is unknown but probably relates to the sudden decrease in effort, loss of the heat produced by the fetus, decrease in intraabdominal pressure, and fetal blood cells entering the maternal circulation. The chill lasts for about 20 minutes and subsides spontaneously. A warm blanket, hot drink, or soup may help relieve the chill and make the woman more comfortable. Placing the baby on her abdomen may result in transfer of heat and make her feel even colder. Reassurance is appropriate after the blanket is provided. Validation of an expected physical response to the birthing process results in a delay of care and is unnecessary. PTS: 1 DIF: Cognitive Level: Application REF: 214

18. The primary difference between the labor of a nullipara and that of a multipara is: a. total duration of labor. b. level of pain experienced. c. amount of cervical dilation. d. sequence of labor mechanisms.

ANS: A Multiparas usually labor more quickly than nulliparas, making the total duration of their labor shorter. The level of pain is individual to the woman, not the number of labors she has experienced. Cervical dilation is the same for all labors. The sequence of labor mechanisms is the same with all labors. PTS: 1 DIF: Cognitive Level: Analysis REF: 214

A woman with a known heroin habit is admitted in early labor. Which drug is contraindicated with opiate-dependent patients? a. Nalbuphine (Nubain) b. Hydroxyzine (Vistaril) c. Promethazine (Phenergan) d. Diphenhydramine (Benadryl)

ANS: A Nalbuphine may precipitate withdrawal if given to an opiate-dependent woman. Hydroxyzine is an antihistamine with antiemetic effects. Promethazine usually relieves nausea and vomiting. Diphenhydramine is commonly used to relieve pruritus from epidural narcotics.

When assessing a multiparous woman who has just given birth to an 8-pound boy, the nurse notes that the woman's fundus is firm and has become globular in shape. A gush of dark red blood comes from her vagina. What is the nurse's assessment of the situation? a.The placenta has separated. b.A cervical tear occurred during the birth. c.The woman is beginning to hemorrhage. d.Clots have formed in the upper uterine segment.

ANS: A Placental separation is indicated by a firmly contracting uterus, a change in the uterus from a discoid to a globular ovoid shape, a sudden gush of dark red blood from the introitus, an apparent lengthening of the umbilical cord, and a finding of vaginal fullness. Cervical tears that do not extend to the vagina result in minimal blood loss. Signs of hemorrhage are a boggy uterus, bright red vaginal bleeding, alterations in vital signs, pallor, lightheadedness, restlessness, decreased urinary output, and alteration in the level of consciousness. If clots have formed in the upper uterine segment, then the nurse would expect to find the uterus boggy and displaced to the side.

Which alterations in the perception of pain by a laboring client should the nurse understand? a. Sensory pain for nulliparous women is often greater than for multiparous women during early labor. b. Affective pain for nulliparous women is usually less than for multiparous women throughout the first stage of labor. c. Women with a history of substance abuse experience more pain during labor. d. Multiparous women have more fatigue from labor and therefore experience more pain.

ANS: A Sensory pain is greater for nulliparous women because their reproductive tract structures are less supple. Affective pain is greater for nulliparous women during the first stage but decreases for both nulliparous and multiparous during the second stage. Women with a history of substance abuse experience the same amount of pain as those without such a history. Nulliparous women have longer labors and therefore experience more fatigue.

Which technique is an adequate means of controlling the birth of the fetal head during delivery in a vertex presentation? a.Ritgen maneuver b.Fundal pressure c.Lithotomy position d.De Lee apparatus

ANS: A The Ritgen maneuver extends the head during the actual birth and protects the perineum. Gentle, steady pressure against the fundus of the uterus facilitates vaginal birth. The lithotomy position has been commonly used in Western cultures, partly because it is convenient for the health care provider. The De Lee apparatus is used to suction fluid from the infant's mouth.

16. A laboring client asks the nurse how she will know that the contraction is at its peak. The nurse explains that the contraction peaks during which stage of measurement? a. The acme b. The interval c. The increment d. The decrement

ANS: A The acme is the peak or period of greatest strength during the middle of a contraction cycle. The interval is the period between the end of the contraction and the beginning of the next. The increment is the beginning of the contraction until it reaches the peak. The decrement occurs after the peak until the contraction ends. PTS: 1 DIF: Cognitive Level: Application REF: 196, 197 OBJ: Nursing Process Step: Implementation

What is the most critical nursing action in caring for the newborn immediately after the birth? a.Keeping the airway clear b.Fostering parent-newborn attachment c.Drying the newborn and wrapping the infant in a blanket d.Administering eye drops and vitamin K

ANS: A The care given immediately after the birth focuses on assessing and stabilizing the newborn. Although fostering parent-newborn attachment is an important task for the nurse, it is not the most critical nursing action in caring for the newborn immediately after birth. The care given immediately after birth focuses on assessing and stabilizing the newborn. The nursing activities are (in order of importance) to maintain a patent airway, to support respiratory effort, and to prevent cold stress by drying the newborn and covering him or her with a warmed blanket or placing the newborn under a radiant warmer. After the newborn has been stabilized, the nurse assesses the newborn's physical condition, weighs and measures the newborn, administers prophylactic eye ointment and a vitamin K injection, affixes an identification bracelet, wraps the newborn in warm blankets, and then gives the newborn to the partner or to the mother of the infant.

The nurse should be aware of which information related to a woman's intake and output during labor? a.Traditionally, restricting the laboring woman to clear liquids and ice chips is being challenged because regional anesthesia is used more often than general anesthesia. b.Intravenous (IV) fluids are usually necessary to ensure that the laboring woman stays hydrated. c.Routine use of an enema empties the rectum and is very helpful for producing a clean, clear delivery. d.When a nulliparous woman experiences the urge to defecate, it often means birth will quickly follow.

ANS: A Women are awake with regional anesthesia and are able to protect their own airway, which reduces the worry over aspiration. Routine IV fluids during labor are unlikely to be beneficial and may be harmful. The routine use of an enema is, at best, ineffective and may be harmful. Having the urge to defecate followed by the birth of her fetus is true for a multiparous woman but not for a nulliparous woman.

The nurse is providing care to a patient in the active phase of the first stage of labor. The patient is crying out loudly with each contraction. What is the nurse's priority action for this patient? a. Ask the patient's labor coach if this is a usual expression of pain for her. b. Refer to the patient's chart to determine any orders for pain medication. c. Tell the patient that she is disturbing the other laboring patients on the unit. d. Encourage the patient to try to suppress her noisiness during contractions

ANS: A Women should be encouraged to express themselves in any way they find comforting, and the diversity of their expressions must be respected. Loud and vigorous expression may be a woman's personal pain coping mechanism, whereas a quiet woman may need medication relief but feels the need to remain stoic. Accepting a woman's individual response to labor and pain promotes a therapeutic relationship. Restraint is difficult because noisy women are challenging to work with and may disturb others.

Which of the following is the priority intervention for the client in a left side-lying position whose monitor strip shows a deceleration that extends beyond the end of the contraction? a. Administer O2 at 8 to 10 L/min. b. Decrease the IV rate to 100 mL/hr. c. Reposition the ultrasound transducer. d. Perform a vaginal exam to assess for cord prolapse.

ANS: A A deceleration that returns to baseline after the end of the contraction is a late deceleration caused by placental perfusion problems. Administering oxygen will increase the client's blood oxygen saturation, making more oxygen available to the fetus. Decreasing the IV rate, repositioning the ultrasound transducer, and performing a vaginal exam to assess for cord prolapse are not effective interventions to improve fetal oxygenation.

Which client is a candidate for internal monitoring with an intrauterine pressure catheter? a. Obese client whose contractions are 3 to 6 minutes apart, lasting 20 to 50 seconds b. Gravida 1, para 0, whose contractions are 2 to 3 minutes apart, lasting 60 seconds c. Multigravida whose contractions are 2 minutes apart, lasting 60 to 70 seconds d. Gravida 2, para 1, in latent phase whose contractions are irregular and mild

ANS: A A thick layer of abdominal fat absorbs energy from uterine contractions, reducing their apparent intensity on the monitor strip. Contraction patterns of 2 to 3 minutes lasting 60 seconds and every 2 minutes lasting 60 to 70 seconds indicate accurate measurement of uterine activity. Irregular and mild contractions are common in the latent phase.

The nurse thoroughly dries the infant immediately after birth primarily to: a. reduce heat loss from evaporation. b. stimulate crying and lung expansion. c. increase blood supply to the hands and feet. d. remove maternal blood from the skin surface.

ANS: A Infants are wet with amniotic fluid and blood at birth, which accelerates evaporative heat loss. Rubbing the infant does stimulate crying but is not the main reason for drying the infant. The main purpose of drying the infant is to prevent heat loss. Drying the infant after birth does not remove all of the maternal blood.

Which assessment finding could indicate hemorrhage in the postpartum patient? a. Elevated pulse rate b. Elevated blood pressure c. Firm fundus at the midline d. Saturation of two perineal pads in 4 hours

ANS: A An increasing pulse rate is an early sign of excessive blood loss. If the blood volume were diminishing, the blood pressure would decrease. A firm fundus indicates that the uterus is contracting and compressing the open blood vessels at the placental site. Saturation of one pad within the first hour is the maximum normal amount of lochial flow. Two pads within 4 hours is within normal limits.

Which can be determined only by electronic fetal monitoring? a. Variability b. Tachycardia c. Bradycardia d. Fetal response to contractions

ANS: A Beat-to-beat variability cannot be determined by auscultation because auscultation provides only an average fetal heart rate (FHR) as it fluctuates. Tachycardia can be determined by any of the FHR monitoring techniques. Bradycardia can be determined by any of the FHR monitoring techniques. The fetal response to the contractions is usually noted by an increase or decrease in fetal heart rate. These can be determined by any of the FHR monitoring techniques.

The nurse is reviewing an electronic fetal monitor tracing from a patient in active labor and notes the fetal heart rate gradually drops to 20 beats per minute (bpm) below the baseline and returns to the baseline well after the completion of the patient's contractions. How will the nurse document these findings? a. Late decelerations b. Early decelerations c. Variable decelerations d. Proximal decelerations

ANS: A Late decelerations are similar to early decelerations in the degree of FHR slowing and lowest rate (30 to 40 bpm) but are shifted to the right in relation to the contraction. They often begin after the peak of the contraction. The FHR returns to baseline after the contraction ends. The early decelerations mirror the contraction, beginning near its onset and returning to the baseline by the end of the contraction, with the low point (nadir) of the deceleration occurring near the contraction's peak. The rate at the lowest point of the deceleration is usually no lower than 30 to 40 bpm from the baseline. Conditions that reduce flow through the umbilical cord may result in variable decelerations. These decelerations do not have the uniform appearance of early and late decelerations. Their shape, duration, and degree of fall below baseline rate vary. They fall and rise abruptly (within 30 seconds) with the onset and relief of cord compression, unlike the gradual fall and rise of early and late decelerations. Proximal decelerations is not a recognized term.

The nurse is monitoring a client in labor and notes this fetal heart rate pattern on the electronic fetal monitoring strip (see figure). Which is the most appropriate nursing action? a. Administer oxygen with a face mask at 8 to 10 L/min. b. Reposition the fetal monitor ultrasound transducer. c. Assist the client to the bathroom to empty her bladder . d. Continue to monitor the client and fetal heart rate patterns.

ANS: A Late decelerations are similar to early decelerations in the degree of FHR slowing and lowest rate (30 to 40 bpm) but are shifted to the right in relation to the contraction. They often begin after the peak of the contraction. They reflect possible impaired placental exchange (uteroplacental insufficiency). Administration of 100% oxygen through a snug face mask makes more oxygen available for transfer to the fetus. A commonly suggested rate is 8 to 10 L/min. The pattern is nonreassuring so repositioning the fetal ultrasound transducer, assisting the client to the bathroom, or continuing to monitor the pattern will not correct the problem.

What is the rationale for the administration of an oxytocic (e.g., Pitocin, Methergine) after expulsion of the placenta? a.To relieve pain b.To stimulate uterine contraction c.To prevent infection d.To facilitate rest and relaxation

ANS: B Oxytocics stimulate uterine contractions, which reduce blood loss after the third stage of labor. Oxytocics are not used to treat pain, do not prevent infection, and do not facilitate rest and relaxation.

9. The nurse auscultates the fetal heart rate and determines a rate of 152 bpm. Which nursing intervention is appropriate? a. Inform the mother that the rate is normal. b. Reassess the fetal heart rate in 5 minutes because the rate is too high. c. Report the fetal heart rate to the physician or nurse-midwife immediately. d. Tell the mother that she is going to have a boy because the heart rate is fast.

ANS: A The FHR is within the normal range, so no other action is indicated at this time. The FHR is within the expected range; reassessment should occur, but not in 5 minutes. The FHR is within the expected range; no further action is necessary at this point. The gender of the baby cannot be determined by the FHR.

The nurse observes the following data on an electronic fetal monitor attached to a client in the active phase of the first stage of labor: fetal heart rate baseline, 125 to 140 bpm, three accelerations over the course of 20 minutes, moderate variability. What is the priority action based on these findings? a. Document the findings. b. Contact the health care provider. c. Increase the rate of the existing IV to 200 mL/hr as per the standing prescription. d. Place oxygen via a rebreather mask at 10 L/min as per the standing prescription.

ANS: A The findings are all within normal limits for the laboring client. Accelerations are usually a reassuring sign. Normal fetal heart rate is 110 to 160 bpm and of moderate variability; amplitude range of 6 to 25 bpm is desirable. No intervention is required because the pattern suggests that the fetus has adequate reserves to tolerate intrapartum stressors.

The nurse is preparing to initiate intravenous (IV) access on a patient in the active phase of labor. Which size IV cannula is best for this patient? a. 18-gauge b. 20-gauge c. 22-gauge d. 24-gauge

ANS: A The larger the number, the smaller the diameter of the cannula. The nurse should select the largest bore cannula possible. IV access is initiated for hydration prior to epidural placement and for use in an emergency. Both require the rapid administration of fluid, which is most easily accomplished with a large bore cannula.

In which situation would a baseline fetal heart rate of 160 to 170 bpm be considered a normal finding? a. The fetus is at 30 weeks of gestation. b. The mother has a history of fast labors. c. The mother has been given an epidural block. d. The mother has mild preeclampsia but is not in labor.

ANS: A The normal preterm fetus may have a baseline rate slightly higher than the term fetus because of an immature parasympathetic nervous system that does not yet exert a slowing effect on the fetal heart rate (FHR). Fast labors should not alter the FHR normally. Any change in the FHR with an epidural is not considered an expected outcome. Preeclampsia should not cause a normal elevation of the FHR.

If a woman's fundus is soft 30 minutes after birth, the nurse's first response should be to: a. massage the fundus. b. take the blood pressure. c. notify the physician or nurse-midwife. d. place the woman in Trendelenburg position.

ANS: A The nurse's first response should be to massage the fundus to stimulate contraction of the uterus to compress open blood vessels at the placental site, limiting blood loss. The blood pressure is an important assessment to determine the extent of blood loss but is not the top priority. Notification should occur after all nursing measures have been attempted with no favorable results. The Trendelenburg position is contraindicated for this woman at this point. This position would not allow for appropriate vaginal drainage of lochia. The lochia remaining in the uterus would clot and produce further bleeding.

When using the second Leopold's maneuver in fetal assessment, the nurse would palpate (the): a. both sides of the maternal abdomen. b. lower abdomen above the symphysis pubis. c. both upper quadrants of the maternal abdomen . d. lower abdomen for flexion of the presenting part.

ANS: A The second Leopold's maneuver involves determining the location of the fetal back and is performed by palpating both sides of the maternal abdomen. Palpating the lower abdomen above the symphysis pubis is the third maneuver. Palpating the upper quadrants of the maternal abdomen is the first maneuver. Palpating the lower abdomen for flexion of the presenting part is the fourth maneuver.

A patient at 41 weeks' gestation is undergoing an induction of labor with an IV administration of oxytocin (Pitocin). The fetal heart rate starts to demonstrate a recurrent pattern of late decelerations with moderate variability. What is the nurse's priority action? a. Stop the infusion of Pitocin. b. Reposition the patient from her right to her left side. c. Perform a vaginal exam to assess for a prolapsed cord. d. Prepare the patient for an emergency cesarean section.

ANS: A There are multiple reasons for late decelerations. Address the probable cause first, such as uterine hyperstimulation with Pitocin, to alleviate the outcome of late decelerations. Repositioning can increase oxygenation to the fetus but does not address the cause of the problem. Variable decelerations are more often seen with a prolapsed cord. In the presence of moderate variability, the fetus continues to have adequate oxygen reserves. The presence of two or more nonreassuring fetal heart rate patterns increases the level of concern.

Which maternal condition should be considered a contraindication for the application of internal monitoring devices? a. Unruptured membranes b. Cervix dilated to 4 cm c. Fetus has known heart defect d. External monitors currently being used

ANS: A To apply internal monitoring devices, the membranes must be ruptured. Cervical dilation of 4 cm would permit the insertion of fetal scalp electrodes and an intrauterine catheter. A compromised fetus should be monitored with the most accurate monitoring devices. The external monitor can be discontinued after the internal ones are applied.

36. The nurse is planning care for a client during the fourth stage of labor. Which interventions should the nurse plan to implement? (Select all that apply.) a. Offer the client a warm blanket. b. Place an ice pack on the perineum. c. Massage the uterus if it is boggy. d. Delay breastfeeding until the client is rested. e. Explain to the client that the lochia will be light pink in color.

ANS: A, B, C The fourth stage of labor lasts from the birth of the placenta through the first 1 to 4 hours after birth. Many women are chilled after birth. A warm blanket, hot drink, or soup may help relieve the chill and make the woman more comfortable. Localized discomfort from birth trauma such as lacerations, episiotomy, edema, or hematoma is evident as the effects of local and regional anesthetics diminish. Ice packs on the perineum limit this edema and hematoma formation. A soft (boggy) uterus and increasing uterine size are associated with postpartum hemorrhage because large blood vessels at the placenta site are not compressed. The uterus should be massaged if it is not firm. The fourth stage is the best time to initiate breastfeeding if maternal and infant problems are absent. The vaginal drainage after childbirth is called lochia. The three stages are lochia rubra, lochia serosa, and lochia alba. Lochia rubra, consisting mostly of blood, is present in the fourth stage of labor. The color of the lochia will be bright red not pink. PTS: 1 DIF: Cognitive Level: Application REF: 214

A laboring client is 10 cm dilated but does not feel the urge to push. The nurse understands that according to laboring down, the advantages of waiting until an urge to push are which of the following? (Select all that apply.) a. Less maternal fatigue b. Less birth canal injuries c. Decreased pushing time d. Faster descent of the fetus e. An increase in frequency of contractions

ANS: A, B, C Delayed pushing has been shown to result in less maternal fatigue and decreased pushing time. Pushing vigorously sooner than the onset of the reflexive urge may contribute to birth canal injury because her vaginal tissues are stretched more forcefully and rapidly than if she pushed spontaneously and in response to her body's signals. A brief slowing of contractions often occurs at the beginning of the second stage.

Which interventions are required following an amniotomy procedure? (Select all that apply.) a. Notation related to amount of fluid expelled b. Color and consistency of fluid c. Fetal heart rate d. Maternal blood pressure e. Maternal heart rate

ANS: A, B, C Following amniotomy (AROM), observation and documentation of the amount of fluid, color and consistency, and fetal heart rate should be done. Maternal assessments related to blood pressure and heart rate are not required.

Women who have participated in childbirth education classes often bring a birth plan with them to the hospital. Which items might this plan include? (Select all that apply.) a.Presence of companions b.Clothing to be worn c.Care and handling of the newborn d.Medical interventions e.Date of delivery

ANS: A, B, C, D The presence of companions, clothing to be worn, care and handling of the newborn, medical interventions, and environmental modifications all might be included in the couple's birth plan. Other items include the presence of nonessential medical personnel (students), labor activities such as the tub or ambulation, preferred comfort and relaxation methods, and any cultural or religious requirements. The expected date of delivery would not be part of a birth plan unless the client is scheduled for an elective cesarean birth.

Which medications could potentially cause hyperstimulation of the uterus during labor? (Select all that apply.) a. Oxytocin (Pitocin) b. Misoprostol (Cytotec) c. Dinoprostone (Cervidil) d. Methylergonovine maleate (Methergine)

ANS: A, B, C, D Oxytocin, misoprostol, and dinoprostone fall under the general category of uterine stimulants. Cytotec and Cervidil are prostaglandins. Methergine is an ergot alkaloid.

Emergency conditions during labor that would require immediate nursing intervention can arise with startling speed. Which situations are examples of such an emergency? (Select all that apply.) a.Nonreassuring or abnormal FHR pattern b.Inadequate uterine relaxation c.Vaginal bleeding d.Prolonged second stage e.Prolapse of the cord

ANS: A, B, C, E A nonreassuring or abnormal FHR pattern, inadequate uterine relaxation, vaginal bleeding, infection, and cord prolapse all constitute an emergency during labor that requires immediate nursing intervention. A prolonged second stage of labor after the upper limits for duration is reached. This is 3 hours for nulliparous women and 2 hours for multiparous women.

When evaluating the client's progress, the nurse knows that four of the five fetal factors that interact to regulate the heart rate are (select all that apply): a. baroreceptors. b. adrenal glands. c. chemoreceptors. d. uterine activity. e. autonomic nervous system.

ANS: A, B, C, E The sympathetic and parasympathetic branches of the autonomic nervous system are balanced forces that regulate FHR. Sympathetic stimulation increases the heart rate, whereas parasympathetic responses, through stimulation of the vagus nerve, reduce the FHR and maintain variability. The baroreceptors stimulate the vagus nerve to slow the FHR and decrease the blood pressure. These are located in the carotid arch and major arteries. The chemoreceptors are cells that respond to changes in oxygen, carbon dioxide, and pH. They are found in the medulla oblongata and aortic and carotid bodies. The adrenal medulla secretes epinephrine and norepinephrine in response to stress, causing accelerations in FHR. Hypertonic uterine activity can reduce the time available for the exchange of oxygen and waste products; however, this is a maternal factor. The fifth fetal factor is the central nervous system. The fetal cerebral cortex causes the heart rate to increase during fetal movement and decrease when the fetus sleeps.

37. Which should the nurse expect to assess in the third stage of labor that indicates the placenta has separated from the uterine wall? (Select all that apply.) a. A gush of blood appears. b. The uterus rises upward in the abdomen. c. The fundus descends below the umbilicus. d. The cord descends further from the vagina. e. The uterus becomes boggy and soft, with an elongated shape.

ANS: A, B, D Four signs suggest placenta separation. The uterus has a spherical shape. The uterus rises upward in the abdomen as the placenta descends into the vagina and pushes the fundus upward. The cord descends further from the vagina. A gush of blood appears as blood trapped behind the placenta is released. The fundus rises upward above the umbilicus. A boggy uterus with an elongated shape would not be expected. PTS: 1 DIF: Cognitive Level: Analysis REF: 214

35. The nurse who elects to practice in the area of obstetrics often hears discussion regarding the four Ps. What are the four Ps that interact during childbirth? (Select all that apply.) a. Powers b. Passage c. Position d. Passenger e. Psyche

ANS: A, B, D, E · Powers: The two powers of labor are uterine contractions and pushing efforts. During the first stage of labor, through full cervical dilation, uterine contractions are the primary force moving the fetus through the maternal pelvis. At some point after full dilation, the woman adds her voluntary pushing efforts to propel the fetus through the pelvis. · Passage: The passage for birth of the fetus consists of the maternal pelvis and its soft tissues. The bony pelvis is more important to the successful outcome of labor because bones and joints do not yield as readily to the forces of labor. · Passenger: This is the fetus plus the membranes and placenta. Fetal lie, attitude, presentation, and position are all factors that affect the fetus as passenger. · Psyche: The psyche is a crucial part of childbirth. Marked anxiety, fear, or fatigue decreases the woman's ability to cope. Position is not one of the four Ps. PTS: 1 DIF: Cognitive Level: Analysis REF: 199

34. A client asks the nurse how she can tell if labor is real? What should the nurse give as an explanation? (Select all that apply.) a. In true labor, the cervix begins to dilate. b. In true labor, the contractions are felt in the abdomen and groin. c. In true labor, contractions often resemble menstrual cramps during early labor. d. In true labor, contractions are inconsistent in frequency, duration, and intensity in the early stages. e. In true labor your contractions tend to increase in frequency, duration, and intensity with walking.

ANS: A, C, E In true labor, the cervix begins to dilate, contractions often resemble menstrual cramps in the early stage, and labor contractions increase in frequency, duration, and intensity with walking. False labor contractions are felt in the abdomen and groin and the contractions are inconsistent in frequency, duration, and intensity. PTS: 1 DIF: Cognitive Level: Application REF: 208

33. A 28-year-old gravida 1, para 0 client who is at term calls the labor and birth unit stating that she thinks she is in labor. She states that she does have some vaginal discharge and feels wet but it is not bloody in nature. She relates a contraction pattern that is irregular, ranging from 5 to 7 minutes and lasting 30 seconds. What questions would be used during the process of phone triage by the nurse? (Select all that apply.) a. Ask her if her if she thinks that her membranes have ruptured. b. Ask her if she has any evidence of bloody show. c. Have her keep monitoring her contraction pattern and call you back if they become more regular. d. Ask her when her she has her next scheduled visit with her health care provider. e. Tell her to come into the hospital for evaluation.

ANS: A, E The cornerstone of obstetric triage is reassurance of maternal-fetal well-being. Thus, in view of the assessment data that the client provided, the nurse should ascertain membrane status and ask the client to come in for evaluation. The client has already indicated that the vaginal discharge was not bloody in nature. Having the client continue to monitor at home would not provide assurance of maternal-fetal well-being. Asking the client about the next scheduled physician visit does not address current health concerns of impending labor. PTS: 1 DIF: Cognitive Level: Analysis REF: 208

A woman who is 39 weeks pregnant expresses fear about her impending labor and how she will manage. What is the nurse's ideal response? a."Don't worry about it. You'll do fine." b."It's normal to be anxious about labor. Let's discuss what makes you afraid." c."Labor is scary to think about, but the actual experience isn't." d."You can have an epidural. You won't feel anything."

ANS: B "It's normal to be anxious about labor. Let's discuss what makes you afraid" is a statement that allows the woman to share her concerns with the nurse and is a therapeutic communication tool. "Don't worry about it. You'll do fine" negates the woman's fears and is not therapeutic. "Labor is scary to think about, but the actual experience isn't" negates the woman's fears and offers a false sense of security. To suggest that every woman can have an epidural is untrue. A number of criteria must be met before an epidural is considered. Furthermore, many women still experience the feeling of pressure with an epidural.

A multipara's labor plan includes the use of jet hydrotherapy during the active phase of labor. What is the priority patient assessment prior to assisting the patient with this request? a. Maternal pulse b. Maternal temperature c. Maternal blood pressure d. Maternal blood glucose level

ANS: B A shower, tub bath, or whirlpool bath is relaxing and provides thermal stimulation. Several studies have shown benefits of water therapy during labor, including immersion in a tub or whirlpool (jet hydrotherapy, or Jacuzzi). The major concern about immersion therapy has been newborn and postpartum maternal infections caused by microorganisms in the water. Infections can be caused by the woman's own ascending vaginal bacteria or by preexisting organisms in an improperly cleaned tub. However, several studies have not found a significant association between newborn or postpartum maternal infections and the use of immersion hydrotherapy with proper cleaning.

22. On admission to the labor and birth unit, a 38-year-old female, gravida 4, para 3, at term in early labor is found to have a transverse lie on vaginal examination. What is the priority intervention at this time? a. Perform a vaginal exam to denote progress. b. Notify the health care provider. c. Initiate parenteral therapy. d. Apply oxygen via nasal cannula at 8 L/min

ANS: B A transverse lie is considered to be an abnormal presentation so the physician should be notified and the process of a C section as the birth method should be initiated. The information provided relative to transverse lie was found on vaginal exam. At this point, the priority is to prepare for a surgical birth because assessment data also indicate that the client is in early labor; thus, a vaginal birth is not imminent. Although initiating parenteral therapy will be required, it is not the priority at this time. Application of oxygen is not required because there is no evidence of fetal or maternal distress. PTS: 1 DIF: Cognitive Level: Analysis REF: 202

Which statement concerning the third stage of labor is correct? a.The placenta eventually detaches itself from a flaccid uterus. b.An expectant or active approach to managing this stage of labor reduces the risk of complications. c.It is important that the dark, roughened maternal surface of the placenta appears before the shiny fetal surface. d.The major risk for women during the third stage is a rapid heart rate.

ANS: B Active management facilitates placental separation and expulsion, reducing the risk of complications. The placenta cannot detach itself from a flaccid (relaxed) uterus. Which surface of the placenta comes out first is not clinically important. The major risk for women during the third stage of labor is postpartum hemorrhaging.

A client in labor is approaching the transition stage and already has an epidural in place. An additional dose of medication has been prescribed and administered to the client. Which priority intervention should be done by the nurse to help evaluate clinical response to treatment? a. Obtain a pain scale response from the client based on a 0 to 10 scale. b. Document maternal blood pressure and fetal heart rates following medication administration and observe for any variations. c. Document intake and output on the electronic health record (EHR). d. Increase the flow rate of prescribed parenteral fluid to maintain hydration.

ANS: B Association of Women's Health, Obstetric and Neonatal Nurses (AWHONN) evidence-based practice guidelines note that maternal blood pressure and fetal heart tones should be assessed following any bolus of additional medication via the epidural route. Obtaining a pain scale response is not typically used for the laboring client but used for postoperative and/or chronic pain clients. Intake and output should be documented as part of the clinical record but is not the priority intervention based on this client's situation. Increasing the flow rate of parenteral fluids requires a physician's order, and there is no clinical evidence that this is needed. Giving parenteral fluids in excess can lead to fluid retention and fluid volume excess.

Childbirth pain is different from other types of pain in that it is: a. less intense. b. associated with a physiologic process. c. more responsive to pharmacologic management. d. designed to make one withdraw from the stimulus.

ANS: B Childbirth pain is part of a normal process, whereas other types of pain usually signify an injury or illness. Childbirth pain is not less intense than other types of pain. Pain management during labor may affect the course and length of labor. The pain with childbirth is a normal process; it is not caused by the type of injury as when withdrawal from the stimulus occurs.

15. The nurse is assessing a client in the active phase of labor. What should the nurse expect during this phase? a. The client is sociable and excited. b. The client is requesting pain medication. c. The client begins to experience the urge to push. d. The client experiences loss of control and irritability.

ANS: B During the active phase of labor, contraction intensity and discomfort increase to the point where women often request pain medication. Sociability and excitability occur during the latent phase. The urge to push occurs at the end of the transition phase or the second stage of labor. Loss of control and irritability occur during the transition phase of labor. PTS: 1 DIF: Cognitive Level: Application REF: 212

5. Which mechanism of labor occurs when the largest diameter of the fetal presenting part passes the pelvic inlet? a. Extension b. Engagement c. Internal rotation d. External rotation

ANS: B Engagement occurs when the presenting part fully enters the pelvic inlet. Extension occurs when the fetal head meets resistance from the tissues of the pelvic floor and the fetal neck stops under the symphysis. This causes the fetal head to extend. Internal rotation occurs when the fetus enters the pelvic inlet. The rotation allows the longest fetal head diameter to conform to the longest diameter of the maternal pelvis. External rotation occurs after the birth of the head. The head then turns to the side so the shoulders can internally rotate and are positioned with their transverse diameter in the anteroposterior diameter of the pelvic outlet. PTS: 1 DIF: Cognitive Level: Understanding REF: 209

The role of the nurse with regard to informed consent is to: a. Inform the patient about the procedure and have her sign the consent form. b. Act as a patient advocate and help clarify the procedure and the options. c. Call the physician to see the patient. d. Witness the signing of the consent form.

ANS: B Feedback A The physician is responsible for informing the woman of her options, explaining the procedure, and advising the patient about potential risk factors. B Nurses play a part in the informed consent process by clarifying and describing procedures or by acting as the woman's advocate and asking the primary health care provider for further explanations. C The physician must be present to explain the procedure to the patient. However, the nurse's responsibilities go further than simply asking the physician to see the patient. D The nurse may witness the signing of the consent form. However, depending on the state's guidelines, the woman's husband or another hospital health care employee may sign as witness.

Which fetal position may cause the laboring client more back discomfort? a. Left occiput anterior b. Left occiput posterior c. Right occiput anterior d. Right occiput transverse

ANS: B In the left occiput posterior position, each contraction pushes the fetal head against the mother's sacrum, which results in intense back discomfort. Back labor is seen mostly when the fetus is in the posterior position.

28. The examiner indicates to the labor nurse that the fetus is in the left occiput anterior (LOA) position. To facilitate the labor process, how will the nurse position the laboring patient? a. On her back b. On her left side c. On her right side d. On her hands and knees

ANS: B LOA is the desired fetal position for the birthing process. Positioning the patient on her left side will accomplish two objectives: (1) by the use of gravity, the fetus will most likely stay in the LOA position; and (2) increase perfusion of the placenta and increase oxygen to the fetus. Positioning the patient on her back decreases placental perfusion. Positioning on her right may facilitate internal rotation and move the fetus out of the LOA position. The hands and knees position is reserved to decrease cord compression, facilitate the fetus out of a posterior position, or increase oxygenation in the presence of hypoxia. Because none of these conditions are present, there is no need to implement this position. PTS: 1 DIF: Cognitive Level: Analysis REF: 204

According to professional standards (AWHONN, 2007) the nonanesthetist registered nurse caring for a woman with an epidural is permitted to perform all actions except: a. Monitor the status of the woman and fetus b. Initiate epidural anesthesia c. Replace empty infusion bags with the same medication and concentrate d. Stop the infusion and initiate emergency measures

ANS: B Only qualified, licensed anesthesia care providers are permitted to insert a catheter, initiate epidural anesthesia, verify catheter placement, inject medication through the catheter, or alter the medication or medications including type, amount, or rate of infusion. The nonanesthetist nurse is permitted to monitor the status of the woman, the fetus, and the progress of labor. Replacement of the empty infusion bags or syringes with the same medication and concentration is permitted. If the need arises the nurse may stop the infusion, initiate emergency measures, and remove the catheter if properly educated to do so. Complications can require immediate interventions. Nurses must be prepared to provide safe and effective care during the emergency situation.

A first-time mother is concerned about the type of medications she will receive during labor. The client is in a fair amount of pain and is nauseated. In addition, she appears to be very anxious. The nurse explains that opioid analgesics are often used along with sedatives. How should the nurse phrase the rationale for this medication combination? a. "The two medications, together, reduce complications." b. "Sedatives enhance the effect of the pain medication." c. "The two medications work better together, enabling you to sleep until you have the baby." d. "This is what your physician has ordered for you."

ANS: B Sedatives may be used to reduce the nausea and vomiting that often accompany opioid use. In addition, some ataractic drugs reduce anxiety and apprehension and potentiate the opioid analgesic affects. A potentiator may cause two drugs to work together more effectively, but it does not ensure zero maternal or fetal complications. Sedation may be a related effect of some ataractic drugs; however, sedation is not the goal. Furthermore, a woman is unlikely to be able to sleep through transitional labor and birth. Although the physician may have ordered the medication, "This is what your physician has ordered for you" is not an acceptable comment for the nurse to make.

After an emergency birth, the nurse encourages the woman to breastfeed her newborn. What is the primary purpose of this activity? a.To facilitate maternal-newborn interaction b.To stimulate the uterus to contract c.To prevent neonatal hypoglycemia d.To initiate the lactation cycle

ANS: B Stimulation of the nipples through breastfeeding or manual stimulation causes the release of oxytocin and prevents maternal hemorrhage. Breastfeeding facilitates maternal-newborn interaction, but it is not the primary reason a woman is encouraged to breastfeed after an emergency birth. The primary intervention for preventing neonatal hypoglycemia is thermoregulation. Cold stress can result in hypoglycemia. The woman is encouraged to breastfeed after an emergency birth to stimulate the release of oxytocin, which prevents hemorrhaging. Breastfeeding is encouraged to initiate the lactation cycle, but it is not the primary reason for this activity after an emergency birth.

Which description of the phases of the first stage of labor is most accurate? a.Latent: mild, regular contractions; no dilation; bloody show b. Active: moderate, regular contractions; 4 to 7 cm dilation c.Lull: no contractions; dilation stable d.Transition: very strong but irregular contractions; 8 to 10 cm dilation

ANS: B The active phase is characterized by moderate and regular contractions, 4 to 7 cm dilation, and duration of 3 to 6 hours. The latent phase is characterized by mild-to-moderate and irregular contractions, dilation up to 3 cm, brownish-to-pale pink mucus, and duration of 6 to 8 hours. No official "lull" phase exists in the first stage. The transition phase is characterized by strong to very strong and regular contractions, 8 to 10 cm dilation, and duration of 20 to 40 minutes.

14. A client whose cervix is dilated to 5 cm is considered to be in which phase of labor? a. Latent phase b. Active phase c. Second stage d. Third stage

ANS: B The active phase of labor is characterized by cervical dilation of 4 to 7 cm. The latent phase is from the beginning of true labor until 3 cm of cervical dilation. The second stage of labor begins when the cervix is completely dilated until the birth of the baby. The third stage of labor is from the birth of the baby until the expulsion of the placenta. PTS: 1 DIF: Cognitive Level: Understanding REF: 212

25. The assessment finding which indicates that the client is in the active phase of the first stage of labor is: a. 80% effacement. b. dilation of 5 cm. c. presence of bloody show. d. regular contraction every 3 to 4 minutes.

ANS: B The active phase of labor is defined by cervical dilation between 4 to 7 cm. Effacement, bloody show, and regular contractions are not parameters whereby the phases of labor are defined. PTS: 1 DIF: Cognitive Level: Analysis REF: 212

10. Which factor ensures that the smallest anterior-posterior diameter of the fetal head enters the pelvis? a. Station b. Flexion c. Descent d. Engagement

ANS: B The anterior-posterior diameter of the head varies with how much it is flexed. In the most favorable situation, the head is fully flexed and the anterior-posterior diameter is the suboccipitobregmatic, averaging 9.5 cm. The station is the relationship of the fetal presenting part to the level of the ischial spine. Descent is the moving of the fetus through the birth canal. Engagement occurs when the largest diameter of the fetal presenting part has passed the pelvic outlet. PTS: 1 DIF: Cognitive Level: Understanding REF: 209

9. Which event is the best indicator of true labor? a. Bloody show b. Cervical dilation and effacement c. Fetal descent into the pelvic inlet d. Uterine contractions every 7 minutes

ANS: B The conclusive distinction between true and false labor is that contractions of true labor cause progressive change in the cervix. Bloody show can occur before true labor. Fetal descent can occur before true labor. False labor may have contractions that occur this frequently but is usually inconsistent. PTS: 1 DIF: Cognitive Level: Understanding REF: 198

A woman who has a history of sexual abuse may have a number of traumatic memories triggered during labor. She may fight the labor process and react with pain or anger. The nurse can implement a number of care measures to help her client view the childbirth experience in a positive manner. Which intervention is key for the nurse to use while providing care? a.Tell the client to relax and that it won't hurt much. b.Limit the number of procedures that invade her body. c.Reassure the client that, as the nurse, you know what is best. d.Allow unlimited care providers to be with the client

ANS: B The number of invasive procedures such as vaginal examinations, internal monitoring, and IV therapy should be limited as much as possible. The nurse should always avoid words and phrases that may result in the client's recalling the phrases of her abuser (i.e., "Relax, this won't hurt" or "Just open your legs"). The woman's sense of control should be maintained at all times. The nurse should explain procedures at the client's pace and wait for permission to proceed. Protecting the client's environment by providing privacy and limiting the number of staff who observe the client will help to make her feel safe.

The physician has ordered an amnioinfusion for the laboring client. What data supports the use of this therapeutic procedure? a. Presenting part not engaged b. +4 meconium-stained amniotic fluid on artificial rupture of membranes (AROM) c. Breech position of fetus d. Twin gestation

ANS: B Amnioinfusion is a procedure used during labor when cord compression or the detection of gross meconium staining is found in the amniotic fluid. A saline solution is used as an irrigation method through the IUPC (intrauterine pressure catheter).

27. The health care provider for a laboring patient makes the following entry into the patient's record: 3/50%/-1. What instruction will the nurse implement with the patient? a. "You will need to remain in bed attached to the electronic fetal monitor." b. "Breathe with me slowly, in through your nose and out through your mouth." c. "I will begin the administration of 1000 mL of IV fluid so you can have an epidural." d. "Your partner will need to change into scrub attire to attend the imminent birth."

ANS: B This client is in the latent phase of the first stage of labor. Use slow, deep chest breathing patterns early in labor to conserve energy for the upcoming process. There is no mention in the stem that the membranes are ruptured, which may prohibit the patient from ambulating. Ambulating during early labor uses gravity to facilitate fetal descent. This is desired because the head is at -1 station. Epidural placement during early labor may slow down the labor process. There is no indication that birth is imminent because the patient is 3 cm dilated. PTS: 1 DIF: Cognitive Level: Analysis REF: 212

Through a vaginal examination, the nurse determines that a woman is 4 cm dilated. The external fetal monitor shows uterine contractions every to 4 minutes. The nurse reports this as what stage of labor? a.First stage, latent phase b.First stage, active phase c.First stage, transition phase d.Second stage, latent phase

ANS: B This maternal progress indicates that the woman is in the active phase of the first stage of labor. During the latent phase of the first stage of labor, the expected maternal progress is 0 to 3 cm dilation with contractions every 5 to 30 minutes. During the transition phase of the first stage of labor, the expected maternal progress is 8 to 10 cm dilation with contractions every 2 to 3 minutes. During the latent phase of the second stage of labor, the woman is completely dilated and experiences a restful period of "laboring down."

31. A laboring patient states to the nurse, "I have to push!" What is the next nursing action? a. Contact the health care provider. b. Examine the patient's cervix for dilation. c. Review with her how to bear down with contractions. d. Ask her partner to support her head with each push.

ANS: B When the cervix is completely dilated, the head can descend through the pelvis and stimulate the Ferguson, or pushing, reflex. Cervical dilation must first be confirmed because premature pushing efforts may result in cervical edema and corresponding delay in dilation. Once complete dilation has been confirmed, the nurse can notify the health care provider. Teaching positioning and pushing efforts is accomplished once complete dilation has been confirmed. PTS: 1 DIF: Cognitive Level: Application REF: 199

1. The husband of a laboring woman asks the nurse how he can help his wife throughout the first stage of labor. The nurse informs him that in addition to all that he's doing now, he could tell her when the contractions are: a. 2 minutes apart. b. at their acme. c. at their increment. d. at their decrement.

ANS: B When the contraction is most intense, the coach can tell the laboring woman that this contraction will be over soon to help her remain focused. Describing the frequency of the contractions is not usually helpful. The increment occurs as the contraction begins in the fundus and spreads through the uterus. Calling attention to this phase may cause the woman to become tense. The woman does not need anyone to tell her that the contraction is decreasing in intensity. PTS: 1 DIF: Cognitive Level: Understanding REF: 196, 197

Which nursing assessment indicates that a woman who is in the second stage of labor is almost ready to give birth? a. Bloody mucous discharge increases. b. The vulva bulges and encircles the fetal head. c. The membranes rupture during a contraction. d. The fetal head is felt at 0 station during the vaginal examination.

ANS: B A bulging vulva that encircles the fetal head describes crowning, which occurs shortly before birth. Bloody show occurs throughout the labor process and is not an indication of an imminent birth. Rupture of membranes can occur at any time during the labor process and does not indicate an imminent birth. Birth of the head occurs when the station is +4. A 0 station indicates engagement.

Which of the following behaviors would be applicable to a nursing diagnosis of risk for injury in a client who is in labor? a. Length of second-stage labor is 2 hours. b. Client has received an epidural for pain control during the labor process. c. Client is using breathing techniques during contractions to maximize pain relief. d. Client is receiving parenteral fluids during the course of labor to maintain hydration.

ANS: B A client who has received medication during labor is at risk for injury as a result of altered sensorium, so this presentation is applicable to the diagnosis. A length of 2 hours for the second stage of labor is within the range of normal. Breathing techniques help maintain control over the labor process. Fluids administered during the labor process are used to prevent potential fluid volume deficit.

The labor nurse is reviewing the cardinal maneuvers with a group of nursing students. Which maneuver will immediately follow the birth of the baby's head? a. Expulsion b. Restitution c. Internal rotation d. External rotation

ANS: B After the head emerges, it realigns with the shoulders (restitution). External rotation occurs as the fetal shoulders rotate internally, aligning their transverse diameter with the anteroposterior diameter of the pelvic outlet. Expulsion occurs when the baby is completely delivered. Internal rotation occurs prior to birth of the head.

A woman who is gravida 3, para 2, enters the intrapartum unit. The most important nursing assessments are: a. contraction pattern, amount of discomfort, and pregnancy history. b. fetal heart rate, maternal vital signs, and the woman's nearness to birth. c. last food intake, when labor began, and cultural practices the couple desires. d. identification of ruptured membranes, the woman's gravida and para, and her support person.

ANS: B All options describe relevant intrapartum nursing assessments, but the focus assessment has priority. If the maternal and fetal conditions are normal and birth is not imminent, other assessments can be performed in an unhurried manner. Contraction pattern, amount of discomfort, and pregnancy history are important nursing assessments but do not take priority if the birth is imminent. Last food intake, when labor began, and cultural practices the couple desires is an assessment that can occur later in the admission process, if time permits. Identification of ruptured membranes, the woman's gravida and para, and her support person are assessments that can occur later in the admission process if time permits.

Which statement correctly describes the nurse's responsibility related to electronic monitoring? a. Report abnormal findings to the physician before initiating corrective actions. b. Teach the woman and her support person about the monitoring equipment and discuss any of their questions. c. Document the frequency, duration, and intensity of contractions measured by the external device. d. Inform the support person that the nurse will be responsible for all comfort measures when the electronic equipment is in place.

ANS: B Teaching is an essential part of the nurse's role. Corrective actions should be initiated first to correct abnormal findings as quickly as possible. Electronic monitoring will record the contractions and FHR response. The support person should still be encouraged to assist with the comfort measures.

A client at 40 weeks' gestation should be instructed to go to a hospital or birth center for evaluation when she experiences: a. fetal movement. b. irregular contractions for 1 hour. c. a trickle of fluid from the vagina. d. thick pink or dark red vaginal mucus.

ANS: C A trickle of fluid from the vagina may indicate rupture of the membranes, requiring evaluation for infection or cord compression. The lack of fetal movement needs further assessment. Irregular contractions are a sign of false labor and do not require further assessment. Bloody show may occur before the onset of true labor. It does not require professional assessment unless the bleeding is pronounced.

At 5 minutes after birth, the nurse assesses that the neonate's heart rate is 96 bpm, respirations are spontaneous, with a strong cry, body posture is flexed with vigorous movement, reflexes are brisk, and there is cyanosis of the hands and feet. What Apgar score will the nurse assign? a. 7 b. 8 c. 9 d. 10

ANS: B The neonate is assigned a score of 1 for heart rate and color and a score of 2 for respiratory effort, muscle tone, and reflex response, for a combined total of 8.

The nurse notes a concerning fetal heart rate pattern for a patient in active labor. The health care provider has prescribed the placement of a Foley catheter. What priority nursing action will the nurse implement when placing the catheter? a. Place the catheter as quickly as possible. b. Place a small pillow under the patient's left hip. c. Omit the use of a cleansing agent, such as Betadine. d. Set up the catheter tray before positioning the patient.

ANS: B To promote placental function, the nurse can place a small pillow or rolled blanket under the patient's left hip to shift the weight of the uterus off the aorta and inferior vena cava. Catheter placement is a sterile procedure, with very prescribed steps. Placing the catheter quickly might lead to skipping a step and place the patient at risk for infection. Use of a cleansing agent, such as Hibiclens or Betadine, is included in the catheter placement procedure to ensure a sterile area for placement. Setting up the catheter tray before positioning the patient is the standard of care.

What is the most likely cause for this fetal heart rate pattern? a. Administration of an epidural for pain relief during labor b. Cord compression c. Breech position of fetus d. Administration of meperidine (Demerol) for pain relief during labor

ANS: B Variable deceleration patterns are seen in response to head compression or cord compression. A breech presentation would not be likely to cause this fetal heart rate pattern. Similarly, administration of medication and/or an epidural would not cause this fetal heart rate pattern.

A nursing priority during admission of a laboring client who has not had prenatal care is: a. obtaining admission labs. b. identifying labor risk factors. c. discussing her birth plan choices. d. explaining importance of prenatal care.

ANS: B When a client has not had prenatal care, the nurse must determine through interviewing and examination the presence of any pregnancy or labor risk factors, obtain admission labs, and discuss birth plan choices. Explaining the importance of prenatal care can be accomplished after the patient's history has been completed.

When the mother's membranes rupture during active labor, the fetal heart rate should be observed for the occurrence of which periodic pattern? a. Early decelerations b. Variable decelerations c. Nonperiodic accelerations d. Increase in baseline variability

ANS: B When the membranes rupture, amniotic fluid may carry the umbilical cord to a position where it will be compressed between the maternal pelvis and the fetal presenting part, resulting in a variable deceleration pattern. Early declarations are considered reassuring; they are not a concern after rupture of membranes. Accelerations are considered reassuring; they are not a concern after rupture of membranes. Increase in baseline variability is not an expected occurrence after the rupture of membranes.

You are preparing a client for epidural placement by a nurse anesthetist in the LDR. Which interventions should be included in the plan of care? (Select all that apply.) a. Administer a bolus of 500 to 1000 mL of D5 normal saline prior to catheter placement. b. Have ephedrine available at bedside during catheter placement. c. Monitor blood pressure of client frequently during catheter insertion and for the first 15 minutes of epidural administration. d. Insert a Foley catheter prior to epidural catheter placement. e. Monitor the client for hypertension in response to epidural insertion.

ANS: B, C A bolus of nondextrose fluid is recommended prior to epidural administration to prevent maternal hypotension. Ephedrine should be available at the bedside in case maternal hypotension is exhibited. Blood pressure should be monitored frequently during insertion and for the first 15 minutes of therapy. It is not necessary to insert a Foley catheter prior to epidural catheter placement. Hypertension is not a common clinical response to this treatment but hypotension is.

38. The nurse is teaching a group of nursing students about factors that have a role in starting labor. Which should the nurse include in the teaching session? (Select all that apply.) a. Progesterone levels become higher than estrogen levels. b. Natural oxytocin in conjunction with other substances plays a role. c. Stretching, pressure, and irritation of the uterus and cervix increase. d. The secretion of prostaglandins from the fetal membranes decreases.

ANS: B, C Factors that appear to have a role in starting labor include the following: (1) natural oxytocin plays a part in labor's initiation in conjunction with other substances; and (2) stretching, pressure, and irritation of the uterus and cervix increase as the fetus reaches term size. The progesterone levels drop and estrogen levels increase. There is an increase in the secretion of prostaglandins from the fetal membranes. PTS: 1 DIF: Cognitive Level: Application REF: 207

Maternal hypotension is a potential side effect of regional anesthesia and analgesia. What nursing interventions could you use to raise the client's blood pressure? Choose all that apply. a. Place the woman in a supine position. b. Place the woman in a lateral position. c. Increase IV fluids. d. Administer oxygen. e. Perform a vaginal examination.

ANS: B, C, D Nursing interventions for maternal hypotension arising from analgesia or anesthesia include turning the woman to a lateral position, increasing IV fluids, administering oxygen via face mask, elevating the woman's legs, notifying the physician, administering an IV vasopressor, and monitoring the maternal and fetal status at least every 5 minutes until these are stable. Placing the client in a supine position causes venous compression, thereby limiting blood flow to and oxygenation of the placenta and fetus. A sterile vaginal examination has no bearing on maternal blood pressure.

When taking care of a client in labor who is not considered to be at risk, which assessments should be included in the plan of care? (Select all that apply.) a. Check the DTR each shift. b. Monitor and record vital signs frequently during the course of labor. c. Document the FHR pattern, noting baseline and response to contraction patterns. d. Indicate on the EFM tracing when maternal position changes are done. e. Provide food, as tolerated, during the course of labor.

ANS: B, C, D Nursing care of the normal laboring client would include monitoring and documentation of vital signs as part of the labor assessment, documentation the FHR, checking patterns to look for assurance of fetal well-being by evaluating baseline and the fetal response to contraction patterns, and noting any position changes on the monitor tracing to evaluate the fetal response. Providing dietary offerings during the course of labor is not part of the nursing care plan because the introduction of food may lead to nausea and vomiting in response to the labor process and might affect the mode of birth.

The nurse recognizes that fetal scalp stimulation may be prescribed to evaluate the response of the fetus to tactile stimulation. Which conditions contraindicate the use of fetal scalp stimulation? (Select all that apply.) a. Post-term fetus b. Maternal fever c. Placenta previa d. Induction of labor e. Prolonged rupture of membranes

ANS: B, C, E Fetal scalp stimulation is not done when there is maternal fever (possibility of introducing microorganisms into the uterus), placenta previa (placenta overlies the cervix, and hemorrhage is likely), or prolonged rupture of membranes (risk of infection). Fetal scalp stimulation may be used to evaluate a post-term fetus' response to stimulation. It is also used to evaluate a fetus when labor is being induced.

Which interventions should be performed in the birth room to facilitate thermoregulation of the newborn? (Select all that apply.) a. Place the infant covered with blankets in the radiant warmer. b. Dry the infant off with sterile towels. c. Place stockinette cap on infant's head. d. Bathe the newborn within 30 minutes of birth. e. Remove wet linen as needed.

ANS: B, C, E Following birth, the newborn is at risk for hypothermia. Therefore, nursing interventions are aimed at maintaining warmth. Drying the infant off, in addition to maintaining warmth, helps stimulate crying and lung expansion, which helps in the transition period following birth. Placing a cap on the infant's head helps prevent heat loss. Removal of wet linens helps minimize further heat loss caused by exposure. Newborns should not be covered while in a radiant warmer with blankets because this will impede birth of heat transfer. Bathing a newborn should be delayed for at least a few hours so that the newborn temperature can stabilize during the transition period.

Which nursing assessment indicates that a woman who is in second-stage labor is almost ready to give birth? a.Fetal head is felt at 0 station during vaginal examination. b.Bloody mucous discharge increases. c.Vulva bulges and encircles the fetal head. d.Membranes rupture during a contraction.

ANS: C A bulging vulva that encircles the fetal head describes crowning, which occurs shortly before birth. Birth of the head occurs when the station is +4. A 0 station indicates engagement. Bloody show occurs throughout the labor process and is not an indication of an imminent birth. ROM can occur at any time during the labor process and does not indicate an imminent birth.

20. Which assessment finding would cause a concern for a client who had delivered vaginally? a. Estimated blood loss (EBL) of 500 mL during the birth process b. White blood cell count of 28,000 mm3 postbirth c. Client complains of fingers tingling d. Client complains of thirst

ANS: C A client's complaint of fingers tingling may represent respiratory alkalosis due to hyperventilation breathing patterns during labor. As such it requires intervention by the nurse to have the client slow breathing down and restore normal carbon dioxide levels. PTS: 1 DIF: Cognitive Level: Application REF: 198

A client presents to the labor and birth area for emergent birth. Vaginal exam reveals that the client is fully dilated, vertex, +2 station, with ruptured membranes. The client is extremely apprehensive because this is her first childbirth experience and asks for an epidural to be administered now. What is the priority nursing response based on this client assessment? a. Use contact anesthesia for an epidural and prepare the client per protocol. b. Tell the client that she will not need any pain medication because the birth will be over in a matter of minutes and the pain will stop. c. Assist the client with nonpharmacologic methods of pain distraction during this time as you prepare for vaginal birth. d. Call the physician for admitting orders

ANS: C By assisting the client with nonpharmacologic methods of pain distraction, the nurse is focusing on the client's needs while still preparing for vaginal birth. The client presents in an emergent situation with birth being imminent. Thus, there is not enough time to administer an epidural. Telling the client that she will not need any pain medication because the birth will be over soon does not address the client's concerns of apprehension and therefore is not therapeutic. Because this is an emergency birth situation, the nurse should be attending to the client. If needed, another nurse and/or supervisor can contact the physician.

23. An assessment finding that would indicate to the nurse that cervical dilation and/or effacement has occurred is: a. onset of irregular contractions. b. cephalic presentation at 0 station. c. bloody mucus drainage from vagina. d. fetal heart tones (FHTs) present in the lower right quadrant.

ANS: C Cervical dilation and/or effacement results in loss of the mucous plug as well as rupture of small capillaries in the cervix; irregular contractions, cephalic presentation, and FHTs in the lower right quadrant do not indicate the onset of cervical ripening. PTS: 1 DIF: Cognitive Level: Analysis REF: 207

Which clinical effect can occur in the presence of increased maternal pain perception during labor? a. Increase in uterine contractions in response to catecholamine secretion b. Decrease in blood pressure in response to alpha receptors c. Decreased perfusion to the placenta in response to catecholamine secretion d. Increased uterine blood flow, causing increase in maternal blood pressure

ANS: C Decreased perfusion to and from the placenta occurs as result of catecholamine secretion. A decrease in uterine contractions is seen in response to catecholamine secretion. Maternal blood pressure is increased in response to alpha receptors. Decreased uterine blood flow causes an increase in maternal blood pressure.

8. The nurse is assessing the duration of a client's labor contractions. Which action does the nurse implement to assess the duration of labor contractions? a. Assess the strongest intensity of each contraction. b. Assess uterine relaxation between two contractions. c. Assess from the beginning to the end of each contraction. d. Assess from the beginning of one contraction to the beginning of the next.

ANS: C Duration of labor contractions is the average length of contractions from beginning to end. Assessing the strongest intensity of each contraction assesses the strength or intensity of the contractions. Assessing uterine relaxation between two contractions is the interval of the contraction phase. Assessing from the beginning of one contraction to the beginning of the next is the frequency of the contractions. PTS: 1 DIF: Cognitive Level: Application REF: 196, 197

2. The nurse is explaining to a group of nursing students what occurs during active labor as the uterus contracts. Which statement explains the maternal-fetal exchange of oxygen and waste products during a contraction? a. Is not significantly affected b. Increases as blood pressure decreases c. Diminishes as the spiral arteries are compressed d. Continues except when placental functions are reduced

ANS: C During labor contractions, the maternal blood supply to the placenta gradually stops as the spiral arteries supplying the intervillous space are compressed by the contracting uterine muscle. The exchange of oxygen and waste products is affected by contractions. The exchange of oxygen and waste products decreases. The maternal blood supply to the placenta gradually stops with contractions. PTS: 1 DIF: Cognitive Level: Application REF: 199

Which clinical finding indicates that the client has reached the second stage of labor? a.Amniotic membranes rupture. b.Cervix cannot be felt during a vaginal examination. c.Woman experiences a strong urge to bear down. d.Presenting part of the fetus is below the ischial spines.

ANS: C During the descent phase of the second stage of labor, the woman may experience an increase in the urge to bear down. The ROM has no significance in determining the stage of labor. The second stage of labor begins with full cervical dilation. Many women may have an urge to bear down when the presenting fetal part is below the level of the ischial spines. This urge can occur during the first stage of labor, as early as with 5 cm dilation.

A woman has requested an epidural for her pain. She is 5 cm dilated and 100% effaced. The baby is in a vertex position and is engaged. The nurse increases the woman's intravenous fluid for a preprocedural bolus. She reviews her laboratory values and notes that the woman's hemoglobin is 12 g/dl, hematocrit is 38%, platelets are 67,000, and white blood cells (WBCs) are 12,000/mm3. Which factor would contraindicate an epidural for the woman? a. She is too far dilated. b. She is anemic. c. She has thrombocytopenia. d. She is septic

ANS: C Feedback A Typically epidural analgesia/anesthesia is used in the laboring woman when a regular labor pattern has been achieved, as evidenced by progressive cervical change. B The laboratory values show that the woman's hemoglobin and hematocrit are in the normal range. C The platelet count indicates thrombocytopenia (low platelets), which is a contraindication to epidural analgesia/anesthesia. D The laboratory values show a slight increase in the WBC count; this is not uncommon in laboring women.

A woman who is 6 cm dilated has the urge to push. The nurse would instruct the woman to: a. Use slow-paced breathing during the contraction b. Hold her breath and push during the contraction c. Blow in short breaths during the contraction d. Use rapid-paced breathing during the contraction

ANS: C If a laboring woman feels the urge to push before the cervix is fully dilated, then she is taught to blow in short breaths to avoid bearing down.

30. The nurse assess a laboring patient's contraction pattern and notes the frequency at every 3 to 4 minutes, duration 50 to 60 sections, and the intensity is moderate by palpation. What is the most accurate documentation for this contraction pattern? a. Stage 1, latent phase b. Stage 2, latent phase c. Stage 1, active phase d. Stage 2, active phase

ANS: C In the active phase of stage 1, contractions are about 2 to 5 minutes apart, with a duration of about 40 to 60 seconds and an intensity that ranges from moderate to strong. During the latent phase of stage 1, the interval between contractions shortens until contractions are about 5 minutes apart. Duration increases to 30 to 40 seconds by the end of the latent phase. During stage 2, latent phase, the woman is resting and preparing to push; she likely has not experienced the Ferguson reflex. She is actively bearing down during the active phase of the second stage. PTS: 1 DIF: Cognitive Level: Understanding REF: 212

What is the primary rationale for the thorough drying of the infant immediately after birth? a.Stimulates crying and lung expansion b.Removes maternal blood from the skin surface c.Reduces heat loss from evaporation d.Increases blood supply to the hands and feet

ANS: C Infants are wet with amniotic fluid and blood at birth, and this accelerates evaporative heat loss. The primary purpose of drying the infant is to prevent heat loss. Although rubbing the infant stimulates crying, it is not the main reason for drying the infant. This process does not remove all the maternal blood.

24. If a notation on the client's health record states that the fetal position is LSP, this means that the: a. head is in the right posterior quadrant of the pelvis. b. head is in the left anterior quadrant of the pelvis. c. buttocks are in the left posterior quadrant of the pelvis. d. buttocks are in the right upper quadrant of the abdomen.

ANS: C LSP explains the position of the fetus in the maternal pelvis. L = left side of the pelvis, S = sacrum (fetus is in breech presentation), P = posterior quadrants of the pelvis. When the head is in the right posterior quadrant of the pelvis, the position is ROP. When the head is in the left anterior quadrant of the pelvis, the position is ROA. When the buttocks are in the upper quadrant of the abdomen, the position would be ROA, ROP, LOA, LOP, LOT, or ROT. PTS: 1 DIF: Cognitive Level: Analysis REF: 204

he method of anesthesia in labor that is considered the safest for the fetus is: a. epidural block. b. pudendal block. c. local infiltration. d. spinal (subarachnoid) block.

ANS: C Local infiltration of the perineum rarely has any adverse effects on the mother or the fetus. With an epidural, pudendal, or spinal block the fetus can be affected by maternal side effects and maternal hypotension.

Where is the point of maximal intensity (PMI) of the FHR located? a.Usually directly over the fetal abdomen b.In a vertex position, heard above the mother's umbilicus c.Heard lower and closer to the midline of the mother's abdomen as the fetus descends and internally rotates d.In a breech position, heard below the mother's umbilicus

ANS: C Nurses should be prepared for the shift. The PMI of the FHR is usually directly over the fetal back. In a vertex position, the PMI of the FHR is heard below the mother's umbilicus. In a breech position, it is heard above the mother's umbilicus.

Which statement by the client will assist the nurse in determining whether she is in true labor as opposed to false labor? a."I passed some thick, pink mucus when I urinated this morning." b."My bag of waters just broke." c."The contractions in my uterus are getting stronger and closer together." d."My baby dropped, and I have to urinate more frequently now."

ANS: C Regular, strong contractions with the presence of cervical change indicate that the woman is experiencing true labor. The loss of the mucous plug (operculum) often occurs during the first stage of labor or before the onset of labor, but it is not the indicator of true labor. Spontaneous rupture of membranes (ROM) often occurs during the first stage of labor, but it is not the indicator of true labor. The presenting part of the fetus typically becomes engaged in the pelvis at the onset of labor, but this is not the indicator of true labor.

Which description of the phases of the second stage of labor is most accurate? a.Latent phase: Feeling sleepy; fetal station 2+ to 4+; duration of 30 to 45 minutes b.Active phase: Overwhelmingly strong contractions; Ferguson reflux activated; duration of 5 to 15 minutes c.Descent phase: Significant increase in contractions; Ferguson reflux activated; average duration varies d.Transitional phase: Woman "laboring down"; fetal station 0; duration of 15 minutes

ANS: C The descent phase begins with a significant increase in contractions; the Ferguson reflex is activated, and the duration varies, depending on a number of factors. The latent phase is the lull or "laboring down" period at the beginning of the second stage and lasts 10 to 30 minutes on average. The second stage of labor has no active phase. The transition phase is the final phase in the second stage of labor; contractions are strong and painful.

The first 1 to 2 hours after birth is sometimes referred to as what? a.Bonding period b.Third stage of labor c.Fourth stage of labor d.Early postpartum period

ANS: C The first 2 hours of the birth are a critical time for the mother and her baby and is often called the fourth stage of labor. Maternal organs undergo their initial readjustment to a nonpregnant state. The third stage of labor lasts from the birth of the baby to the expulsion of the placenta. Bonding will occur over a much longer period, although it may be initiated during the fourth stage of labor.

The uterine contractions of a woman early in the active phase of labor are assessed by an internal uterine pressure catheter (IUPC). The uterine contractions occur every 3 to 4 minutes and last an average of 55 to 60 seconds. They are becoming more regular and are moderate to strong. Based on this information, what would a prudent nurse do next? a.Immediately notify the woman's primary health care provider. b.Prepare to administer an oxytocic to stimulate uterine activity. c.Document the findings because they reflect the expected contraction pattern for the active phase of labor. d.Prepare the woman for the onset of the second stage of labor.

ANS: C The nurse is responsible for monitoring the uterine contractions to ascertain whether they are powerful and frequent enough to accomplish the work of expelling the fetus and the placenta. In addition, the nurse documents these findings in the client's medical record. This labor pattern indicates that the client is in the active phase of the first stage of labor. Nothing indicates a need to notify the primary health care provider at this time. Oxytocin augmentation is not needed for this labor pattern; this contraction pattern indicates that the woman is in active labor. Her contractions will eventually become stronger, last longer, and come closer together during the transition phase of the first stage of labor. The transition phase precedes the second stage of labor, or delivery of the fetus.

Which action is correct when palpation is used to assess the characteristics and pattern of uterine contractions? a.Placing the hand on the abdomen below the umbilicus and palpating uterine tone with the fingertips b.Determining the frequency by timing from the end of one contraction to the end of the next contraction c.Evaluating the intensity by pressing the fingertips into the uterine fundus d.Assessing uterine contractions every 30 minutes throughout the first stage of labor

ANS: C The nurse or primary health care provider may assess uterine activity by palpating the fundal section of the uterus using the fingertips. Many women may experience labor pain in the lower segment of the uterus, which may be unrelated to the firmness of the contraction detectable in the uterine fundus. The frequency of uterine contractions is determined by palpating from the beginning of one contraction to the beginning of the next contraction. Assessment of uterine activity is performed in intervals based on the stage of labor. As labor progresses, this assessment is performed more frequently.

Which statement is true with regard to the type of pain associated with childbirth experience? a. Pain is constant throughout the labor experience. b. Labor pain during childbirth is considered to be an abnormal response. c. Pain associated with childbirth is self-limiting. d. Pain associated with childbirth does not allow for adequate preparation.

ANS: C The pain associated with childbirth is self-limiting in that it typically stops once the child is delivered. Pain is intermittent during the labor experience. Labor pain is considered to be a normal response during childbirth. Pregnant woman can prepare for the expected pain of childbirth by taking prepared childbirth classes and using relaxation techniques during the course of labor.

A woman in the transition phase of labor requests a narcotic analgesic medication for pain relief. The nurse explains that giving a narcotic analgesic medication at this stage of labor will: a. Cause medication given at later stages ineffective b. Have no complications for the mother or baby c. Result in respiratory depression to the newborn d. Speed up labor and increase pain

ANS: C The risk of narcotic analgesics is that they cross the placenta and can cause the infant's breathing to be sluggish. This is most likely if the drug is administered shortly before birth.

A nulliparous woman has just begun the latent phase of the second stage of her labor. The nurse should anticipate which behavior? a.A nulliparous woman will experience a strong urge to bear down. b.Perineal bulging will show. c.A nulliparous woman will remain quiet with her eyes closed between contractions. d.The amount of bright red bloody show will increase

ANS: C The woman is able to relax and close her eyes between contractions as the fetus passively descends. The woman may be very quiet during this phase. During the latent phase of the second stage of labor, the urge to bear down is often absent or only slight during the acme of the contractions. Perineal bulging occurs during the transition phase of the second stage of labor, not at the beginning of the second stage. An increase in bright red bloody show occurs during the descent phase of the second stage of labor.

A 25-year-old gravida 3, para 2 client gave birth to a 9-pound, 7-ounce boy, 4 hours ago after augmentation of labor with oxytocin (Pitocin). She presses her call light, and asks for her nurse right away, stating "I'm bleeding a lot." What is the most likely cause of postpartum hemorrhaging in this client? a.Retained placental fragments b.Unrepaired vaginal lacerations c.Uterine atony d.Puerperal infection

ANS: C This woman gave birth to a macrosomic infant after oxytocin augmentation. Combined with these risk factors, uterine atony is the most likely cause of bleeding 4 hours after delivery. Although retained placental fragments may cause postpartum hemorrhaging, it is typically detected within the first hour after delivery of the placenta and is not the most likely cause of the hemorrhaging in this woman. Although unrepaired vaginal lacerations may also cause bleeding, it typically occurs in the period immediately after birth. Puerperal infection can cause subinvolution and subsequent bleeding that is, however, typically detected 24 hours postpartum.

The process of labor places significant metabolic demands on the obstetric client. Which physiologic findings would be expected? a. Decreased maternal blood pressure as a result of stimulation of alpha receptors b. Uterine vasoconstriction as a result of stimulation of beta receptors c. Increased maternal demand for oxygen d. Increased blood flow to placenta because of catecholamine release

ANS: C With regard to labor, one would expect to see an increase in maternal blood pressure because of stimulation of alpha receptors. Uterine vasoconstriction would occur in response to stimulation of alpha receptors. One would expect to see a decrease in blood flow to the placenta. The maternal metabolic rate is increased during labor, along with an increase in maternal demand for oxygen.

7. Pregnant clients can usually tolerate the normal blood loss associated with childbirth because they have: a. higher hematocrit. b. increased leukocytes. c. increased blood volume. d. a lower fibrinogen level.

ANS: C Women have a significant increase in blood volume during pregnancy. After birth, the additional circulating volume is no longer necessary. The hematocrit decreases with pregnancy because of the high fluid volume. Leukocyte levels increase during labor, but that is not the reason for the toleration of blood loss. Fibrinogen levels increase with pregnancy. PTS: 1 DIF: Cognitive Level: Understanding REF: 198

Which of the following is the priority intervention for a supine client whose monitor strip shows decelerations that begin after the peak of the contraction and return to the baseline after the contraction ends? a. Increase IV infusion. b. Elevate lower extremities. c. Reposition to left side-lying position. d. Administer oxygen per face mask at 4 to 6 L/min.

ANS: C Decelerations that begin at the peak of the contractions and recover after the contractions end are caused by uteroplacental insufficiency. When the client is in the supine position, the weight of the uterus partially occludes the vena cava and descending aorta, resulting in hypotension and decreased placental perfusion. Increasing the IV infusion, elevating the lower extremities, and administering O2 will not be effective as long as the client is in a supine position.

Client is at 38 weeks' gestation, gravida 1, para 0, vaginal exam—4 cm, 100% effaced, +1 station vertex. What is the most likely intervention for this fetal heart rate pattern? a. Continue oxytocin (Pitocin) infusion. b. Contact the anesthesia department for epidural administration. c. Change maternal position. d. Administer Narcan to client and prepare for immediate vaginal delivery.

ANS: C Late decelerations indicate fetal compromise (uteroplacental insufficiency) and are considered to be a significant event requiring immediate assessment and intervention. Of all the options listed, changing maternal position may increase placental perfusion. In the presence of late decelerations, Pitocin infusion should be stopped. Contacting anesthesia for epidural administration will not solve the existing problem of late decelerations. There are no data to support the administration of Narcan and because client is still in early labor, birth is not imminent.

The nurse is preparing to perform Leopold's maneuvers. Why are Leopold's maneuvers used by practitioners? a. To determine the status of the membranes b. To determine cervical dilation and effacement c. To determine the best location to assess the fetal heart rate d. To determine whether the fetus is in the posterior position

ANS: C Leopold's maneuvers are often performed before assessing the fetal heart rate (FHR). These maneuvers help identify the best location to obtain the FHR. A Nitrazine or ferning test can be performed to determine the status of the fetal membranes. Dilation and effacement are best determined by vaginal examination. Assessment of fetal position is more accurate with vaginal examination.

The nurse notes that a client who has given birth 1 hour ago is touching her infant with her fingertips and talking to him softly in high-pitched tones. Based on this observation, which action should the nurse take? a. Request a social service consult for psychosocial support. b. Observe for other signs that the mother may not be accepting of the infant. c. Document this evidence of normal early maternal-infant attachment behavior. d. Determine whether the mother is too fatigued to interact normally with her infant.

ANS: C Normal early maternal-infant behaviors are tentative and include fingertip touch, eye contact, and using a high-pitched voice when talking to the infant. There is no indication at this point that a social service consult is necessary. The signs are of normal attachment behavior. These are signs of normal attachment behavior; no other assessment is necessary at this point. The mother may be fatigued but is interacting with the infant in an expected manner.

A woman arrives to the labor and birth unit at term. She is greeted by a staff nurse and a nursing student. The student reviews the initial intake assessment with the staff nurse. Which action will the staff nurse have to correct? a. Obtain a fetal heart rate. b. Determine the estimated due date. c. Auscultate anterior and posterior breath sounds. d. Ask the client when she last had something to eat.

ANS: C On admission to the labor and birth unit, a focused assessment is performed. This includes the following: names of mother and support person(s); name of her physician or nurse-midwife if she had prenatal care; number of pregnancies and prior births, including whether the birth was vaginal or cesarean; status of membranes; expected date of birth; problems during this or other pregnancies; allergies to medications, foods, or other substances; time and type of last oral intake; maternal vital signs and FHR; and pain—location, intensity, factors that intensify or relieve, duration, whether constant or intermittent, and whether the pain is acceptable to the woman. Generally, women of childbearing years are healthy and auscultation of lung sounds can be delayed until the initial intake assessment has been completed.

When a nonreassuring pattern of the fetal heart rate is noted and the client is lying on her left side, which nursing action is indicated? a. Lower the head of the bed. b. Place a wedge under the left hip. c. Change her position to the right side. d. Place the mother in Trendelenburg position.

ANS: C Repositioning on the opposite side may relieve compression on the umbilical cord and improve blood flow to the placenta. Lowering the head of the bed would not be the first position change choice. The woman is already on her left side, so a wedge on that side would not be an appropriate choice. Repositioning to the opposite side is the first intervention. If unsuccessful with improving the FHR pattern, further changes in position can be attempted; the Trendelenburg position might be the choice.

When the deceleration pattern of the fetal heart rate mirrors the uterine contraction, which nursing action is indicated? a. Reposition the client. b. Apply a fetal scalp electrode. c. Record this reassuring pattern. d. Administer oxygen by nasal cannula.

ANS: C The periodic pattern described is early deceleration that is not associated with fetal compromise and requires no intervention. It is a reassuring pattern. Repositioning the client, applying a fetal scalp electrode, or administering oxygen would be interventions done for nonreassuring patterns.

The nurse detects hypotension in a laboring client after an epidural. Which actions should the nurse plan to implement? (Select all that apply.) a. Encourage the client to drink fluids. b. Place the client in a Trendelenburg position. c. Administer a normal saline bolus as prescribed. d. Administer oxygen at 8 to 10 L/min per face mask. e. Administer IV ephedrine in 5- to 10-mg increments as prescribed.

ANS: C, D, E If hypotension occurs after an epidural has been placed, techniques such as a rapid nondextrose IV fluid bolus, maternal repositioning, and oxygen administration are implemented. If those interventions are ineffective, IV ephedrine in 5- to 10-mg increments can be prescribed to promote vasoconstriction to raise the blood pressure. The client in active labor should not be encouraged to drink fluids. In a Trendelenburg position, the body is flat, with the feet elevated. This would not be a position to use for a pregnant client.

The nurse is caring for a laboring patient who develops a fever after she has had her epidural initiated. Fever with no apparent infection may occur in women who have epidural analgesia. The neonate's temperature may be elevated as well, possibly leading to unnecessary treatment for neonatal sepsis. Possible explanations for epidural associated fever without infection include (select all that apply) a. Bladder distention b. Maternal hypotension c. Reduction of heat dissipation d. Vasodilation e. Shivering

ANS: C, D, E Feedback Correct Heat dissipation is reduced as a result of decreased hyperventilation, sweating, and activity after the onset of pain relief. Vasodilation redistributes heat from the core to the periphery of the body, where it is lost to the environment. Shivering often occurs with sympathetic blockade accompanied by a dissociation between warm and cold sensations. In essence, the body believes that the temperature is lower than it actually is and turns up the "thermostat." Incorrect Bladder distention is an anticipated effect of having an epidural. A woman's bladder fills quickly because of the large quantity of IV solution, yet her sensation to void is reduced. Maternal hypotension is an expected side effect of epidural initiation

When a nulliparous woman telephones the hospital to report that she is in labor, what guidance should the nurse provide or information should the nurse obtain? a.Tell the woman to stay home until her membranes rupture. b.Emphasize that food and fluid intake should stop. c.Arrange for the woman to come to the hospital for labor evaluation. d.Ask the woman to describe why she believes she is in labor.

ANS: D Assessment begins at the first contact with the woman, whether by telephone or in person. By asking the woman to describe her signs and symptoms, the nurse can begin her assessment and gather data. The initial nursing activity should be to gather data about the woman's status. The amniotic membranes may or may not spontaneously rupture during labor. The client may be instructed to stay home until the uterine contractions become strong and regular. Before instructing the woman to come to the hospital, the nurse should initiate her assessment during the telephone interview. After this assessment has been made, the nurse may want to discuss the appropriate oral intake for early labor, such as light foods or clear liquids, depending on the preference of the client or her primary health care provider.

A woman who is pregnant for the first time is dilated 3 cm and having contractions every 5 minutes. She is groaning and perspiring excessively; she states that she did not attend childbirth classes. The most important nursing action is to: a. Notify the woman's health care provider b. Administer the prescribed narcotic analgesic c. Assure her that her labor will be over soon d. Assist her with simple breathing and relaxation instructions

ANS: D By reducing tension and stress, focusing and relaxation techniques allow the woman in labor to rest and conserve energy for the task of giving birth. For those who have had no preparation, instruction in simple breathing and relaxation can be given in early labor and often is successful. The nurse can perform many functions in labor and birth independently, such as teaching and support. Pain medication may be an option for this client. However, the initial response of the nurse should include educating the client about her options.The length of labor varies among individuals, but the first stage of labor is the longest. At 3 cm of dilation with contractions every 5 minutes, this woman has a significant amount of labor yet to experience.

3. The nurse is directing an unlicensed assistive personnel (UAP) to take maternal vital signs between contractions. Which statement is the best rationale for assessing maternal vital signs between contractions? a. Vital signs taken during contractions are not accurate. b. During a contraction, assessing fetal heart rate is the priority. c. Maternal blood flow to the heart is reduced during contractions. d. Maternal circulating blood volume increases temporarily during contractions.

ANS: D During uterine contractions, blood flow to the placenta temporarily stops, causing a relative increase in the mother's blood volume, which in turn temporarily increases blood pressure and slows the pulse. Vital signs are altered by contractions but are considered accurate for a period of time. It is important to monitor the fetal response to contractions, but the question is concerned with the maternal vital signs. Maternal blood flow is increased during a contraction. PTS: 1 DIF: Cognitive Level: Application REF: 198

A multiparous woman has been in labor for 8 hours. Her membranes have just ruptured. What is the nurse's highest priority in this situation? a.Prepare the woman for imminent birth. b.Notify the woman's primary health care provider. c.Document the characteristics of the fluid. d.Assess the fetal heart rate (FHR) and pattern.

ANS: D The umbilical cord may prolapse when the membranes rupture. The FHR and pattern should be closely monitored for several minutes immediately after the ROM to ascertain fetal well-being, and the findings should be documented. The ROM may increase the intensity and frequency of the uterine contractions, but it does not indicate that birth is imminent. The nurse may notify the primary health care provider after ROM occurs and the fetal well-being and response to ROM have been assessed. The nurse's priority is to assess fetal well-being. The nurse should document the characteristics of the amniotic fluid, but the initial response is to assess fetal well-being and the response to ROM.

A primigravida at 39 weeks of gestation is observed for 2 hours in the intrapartum unit. The FHR has been normal. Contractions are 5 to 9 minutes apart, 20 to 30 seconds in duration, and of mild intensity. Cervical dilation is 1 to 2 cm and uneffaced (unchanged from admission). Membranes are intact. What disposition would the nurse anticipate? a.Admitted and prepared for a cesarean birth b.Admitted for extended observation c.Discharged home with a sedative d.Discharged home to await the onset of true labor

ANS: D This situation describes a woman with normal assessments who is probably in false labor and will likely not deliver rapidly once true labor begins. No further assessments or observations are indicated; therefore, the client will be discharged along with instructions to return when contractions increase in intensity and frequency. Neither a cesarean birth nor a sedative is required at this time.

To improve placental blood flow immediately after the injection of an epidural anesthetic, the nurse should: a. give the woman oxygen. b. turn the woman to the right side. c. decrease the intravenous infusion rate. d. place a wedge under the woman's right hip.

ANS: D Tilting the woman's pelvis to the left side relieves compression of the vena cava and compensates for a lower blood pressure without interfering with dispersal of the epidural medication. Oxygen administration will not improve placental blood flow. The woman needs to maintain the supine position for proper dispersal of the medication. However, placing a wedge under the hip will relieve compression of the vena cava. The intravenous infusion rate needs to be increased to prevent hypotension.

A client is in early labor, and her nurse is discussing the pain relief options she is considering. The client states that she wants an epidural "no matter what!" What is the nurse's best response? a. "I'll make sure you get your epidural." b. "You may only have an epidural if your physician allows it." c. "You may only have an epidural if you are going to deliver vaginally." d. "The type of analgesia or anesthesia used is determined, in part, by the stage of your labor and the method of birth."

ANS: D To avoid suppressing the progress of labor, pharmacologic measures for pain relief are generally not implemented until labor has advanced to the active phase of the first stage and the cervix is dilated approximately 4 to 5 cm. A plan of care is developed for each woman that addresses her particular clinical and nursing problems. The nurse collaborates with the primary health care provider and the laboring woman in selecting features of care relevant to the woman and her family. The decision whether to use an epidural to relieve labor pain is multifactorial. The nurse should not make a blanket statement guaranteeing the client one pharmacologic option over another until a complete history and physical examination has been obtained. A physician's order is required for pharmacologic options for pain management. However, expressing this requirement is not the nurse's best response. An epidural is an effective pharmacologic pain management option for many laboring women. It can also be used for anesthesia control if the woman undergoes an operative delivery.

Maternity nurses often have to answer questions about the many, sometimes unusual ways people have tried to make the birthing experience more comfortable. For instance, nurses should be aware that: a. Music supplied by the support person has to be discouraged because it could disturb others or upset the hospital routine b. Women in labor can benefit from sitting in a bathtub, but they must limit immersion to no longer than 15 minutes at a time c. Effleurage is permissible, but counterpressure is almost always counterproductive d. Electrodes attached to either side of the spine to provide high-intensity electrical impulses facilitate the release of endorphins

ANS: D Transcutaneous electrical nerve stimulation (TENS) may help and is most useful for lower back pain that occurs during the first stage of labor. Music may be very helpful for reducing tension and certainly can be accommodated by the hospital. Women can stay in a bath as long as they want, although repeated baths with breaks might be more effective than a long soak.Counterpressure can help the woman cope with lower back pain.

Which characteristic of a uterine contraction is not routinely documented? a.Frequency: how often contractions occur b.Intensity: strength of the contraction at its peak c.Resting tone: tension in the uterine muscle d.Appearance: shape and height

ANS: D Uterine contractions are described in terms of frequency, intensity, duration, and resting tone. Appearance is not routinely charted.

A nurse is teaching a childbirth education class. Which information about excessive pain in labor should the nurse include in the session? a. It usually results in a more rapid labor. b. It has no effect on the outcome of labor. c. It is considered to be a normal occurrence. d. It may result in decreased placental perfusion.

ANS: D When experiencing excessive pain, the woman may react with a stress response that diverts blood flow from the uterus and the fetus. Excessive pain may prolong the labor because of increased anxiety in the woman. It may affect the outcome of the labor, depending on the cause and the effect on the woman. Pain is considered normal for labor. However, excessive pain may be an indication of other problems and must be assessed.

The nurse sees a pattern on the fetal monitor that looks similar to early decelerations, but the deceleration begins near the acme of the contraction and continues well beyond the end of the contraction. Which nursing action indicates the proper evaluation of this situation? a. This pattern reflects variable decelerations. No interventions are necessary at this time. b. Document this reassuring fetal heart rate pattern but decrease the rate of the intravenous (IV) fluid. c. Continue to monitor these early decelerations, which occur as the fetal head is compressed during a contraction. d. This deceleration pattern is associated with uteroplacental insufficiency, so the nurse acts quickly to improve placental blood flow and fetal oxygen supply.

ANS: D A pattern similar to early decelerations, but the deceleration begins near the acme of the contraction and continues well beyond the end of the contraction, describes a late deceleration. Oxygen should be given via a snug face mask. Position the client on her left side to increase placental blood flow. Variable decelerations are caused by cord compression. A vaginal examination should be performed to identify this potential emergency. This is not a reassuring pattern, so the IV rate should be increased to increase the mother's blood volume. These are late decelerations, not early; therefore, interventions are necessary.

Which nursing diagnosis would take priority in the care of a primipara client with no visible support person in attendance who has entered the second stage of labor after a first stage of labor lasting 4 hours? a. Fluid volume deficit (FVD) related to fluid loss during labor and birth process b. Fatigue related to length of labor requiring increased energy expenditure c. Acute pain related to increased intensity of contractions d. Anxiety related to imminent birth process

ANS: D A primipara is experiencing the birthing event for the first time and may experience anxiety because of fear of the unknown. It would be important to recognize this because the client is alone in the labor-birth room and will need additional support and reassurance. Although FVD may occur as a result of fluid loss, prospective management of labor clients includes the use of parenteral fluid therapy; the client should be monitored for FVD and, if symptoms warrant, receive intervention. Because the client has been in labor for 4 hours, this is not considered to be a prolonged labor pattern for a primipara client. Although the client may be tired, this nursing diagnosis would not be a priority unless there were other symptoms manifested. Because the client is entering the second stage of labor, she will be allowed to push with contractions. Thus, in terms of pain management, medication will not be administered at this time because of imminent birth.

The gynecologist performs an amniotomy. What will the nurse's role include immediately following the procedure? a. Assessing for ballottement b. Conducting a pH and/or fern test c. Labeling of specimens for chromosomal analysis d. Recording the character and amount of amniotic fluid

ANS: D An amniotomy is a procedure in which the amniotic sac is deliberately ruptured. It is important to note and record the character and amount of amniotic fluid following this procedure. Assessing for ballottement is not indicated. Conducting a pH or fern test is not needed because an amniotomy releases amniotic fluid. An amniocentesis, not an amniotomy, is used to collect a specimen for chromosomal analysis.

Which client has the priority need for fetal monitoring? a. Primigravida at 38 weeks with spontaneous ROM b. Multigravida at 40 weeks with history of 10-hour labors c. Multigravida admitted for repeat elective cesarean section d. Primigravida at 39 weeks with meconium-stained amniotic fluid

ANS: D Meconium-stained amniotic fluid indicates a potential risk factor during labor. Primigravida at 38 weeks with spontaneous ROM, multigravida with a history of 10-hour labors, and multigravida admitted for repeat elective cesarean section do not have potential maternal or fetal risk factors.

Which client at term should go to the hospital or birth center the soonest after labor begins? a. Gravida 2, para 1, who lives 10 minutes away b. Gravida 1, para 0, who lives 40 minutes away c. Gravida 2, para 1, whose first labor lasted 16 hours d. Gravida 3, para 2, whose longest previous labor was 4 hours

ANS: D Multiparous women usually have shorter labors than do nulliparous women. The woman described in option D is multiparous with a history of rapid labors, increasing the likelihood that her infant might be born in uncontrolled circumstances. A gravida 2 would be expected to have a longer labor than the gravida in option C. The fact that she lives close to the hospital allows her to stay home for a longer period of time. A gravida 1 will be expected to have the longest labor. The gravida 2 would be expected to have a longer labor than the gravida 3, especially because her first labor was 16 hours.

The nurse examines a primipara's cervix at 8-9/100%/+2; it is tight against the fetal head. The patient reports a strong urge to bear down. What is the nurse's priority action? a. Palpate her bladder for fullness. b. Assess the frequency and duration of her contractions. c. Determine who will stay with the patient for the birth. d. Encourage the patient to exhale in short breaths with contractions.

ANS: D Teach the woman to exhale in short breaths if pushing is likely to injure her cervix or cause cervical edema. Pushing against a cervix that does not easily yield to pressure from the presenting part may result in cervical edema, which can block labor progress or cause cervical lacerations. A full bladder may impede the progress of labor. Although this is an important nursing action, it does not address the patient's urge to push. This patient is in the transition phase of the first stage of labor. Her contractions will be every 2 to 3 minutes and last 60 to 90 seconds. Determining the frequency and duration of the contractions does not add to the known assessment data for this patient. Determining who will attend the birth, although nice to know, does not address her urge to push.

A primigravida at 39 weeks of gestation is observed for 2 hours in the intrapartum unit. The fetal heart rate has been normal. Contractions are 5 to 9 minutes apart, 20 to 30 seconds in duration, and of mild intensity. Cervical dilation is 1 to 2 cm and uneffaced (unchanged from admission). Membranes are intact. The nurse should expect the client to be: a. discharged home with a sedative. b. admitted for extended observation. c. admitted and prepared for a cesarean birth. d. discharged home to await the onset of true labor.

ANS: D The situation describes a client with normal assessments who is probably in false labor and will probably not deliver rapidly once true labor begins. The client will probably be discharged, but there is no indication that a sedative is needed. These are all indications of false labor; there is no indication that further assessment or observations are indicated. These are all indications of false labor without fetal distress. There is no indication that a cesarean birth is indicated.

In which situation would it be appropriate to obtain a fetal scalp blood sample to establish fetal well-being? a. The fetus has developed tachycardia related to maternal fever. b. The mother has vaginal bleeding, and the baseline fetal heart rate is decreasing. c. The fetal heart tracing on a preterm fetus shows decreased baseline variability. d. The fetal heart tracing shows a persistent pattern of late decelerations, with normal baseline variability.

ANS: D The tracing is nonreassuring, and additional assessment is needed regarding the acid-base status of the fetus. Fetal scalp blood sampling is contraindicated with vaginal bleeding, maternal fever, and a preterm fetus.

The nurse is monitoring a client in labor and notes this fetal heart rate pattern on the electronic fetal monitoring strip (see figure). Which is the most appropriate nursing action? a. Decrease the rate of the IV fluids. b. Document the fetal heart rate pattern. c. Explain to the client that the pattern is reassuring. d. Perform a vaginal exam to detect a prolapsed cord.

ANS: D Variable decelerations do not have the uniform appearance of early and late decelerations. Their shape, duration, and degree of fall below baseline rate vary. They fall and rise abruptly (within 30 seconds) with the onset and relief of cord compression, unlike the gradual fall and rise of early and late decelerations. A vaginal examination may identify a prolapsed cord, which may cause variable decelerations, bradycardia, or both as it is compressed. A vaginal examination also evaluates the woman's labor status, which helps the birth attendant decide if labor should continue. This is a nonreassuring pattern so the IV rate should be increased and an intervention needs to be done, not just documentation.

Which should the nurse recognize as being associated with fetal compromise? a. Active fetal movements b. Fetal heart rate in the 140s c. Contractions lasting 90 seconds d. Meconium-stained amniotic fluid

ANS: D When fetal oxygen is compromised, relaxation of the rectal sphincter allows passage of meconium into the amniotic fluid. Active fetal movement is an expected occurrence. The expected FHR range is 120 to 160 bpm. The fetus should be able to tolerate contractions lasting 90 seconds if the resting phase is sufficient to allow for a return of adequate blood flow.

A fetal pulse oximetry is applied to a fetus. The reading shows an oxygen saturation of 45%. The nurse realizes that this means: a. A low reading; normal should be above 95%. b. A high reading; normal should be between 20% and 30%. c. A normal reading. d. This is not conclusive.

C A normal fetal pulse oximetry reading is 30% to 70%. A normal reading for an adult is between 95% and 100%. The lower reading is considered normal because of the high hemoglobin and hematocrit levels in the fetus compared with those of an adult.

A pregnant client walks into the birthing center complaining of contractions. After getting her to bed, the first thing the nurse should do is: a. Assess the mother's pulse and respirations. b. Gather information about her medical history. c. Assess the fetal heart rate. d. Start an intravenous line.

C Assessment priorities on admission of a labor client are to determine the condition of the mother and fetus and whether birth is imminent. Checking the fetal heart rate is one of the first assessments that should be carried out. Along with assessing the fetus, the nurse should also check the maternal blood pressure and temperature.

In which of the following situations would fetal oxygenation be compromised? a. The mother has been taking Tylenol for mild headaches 2 days prior to the onset of labor. b. The mother regularly sleeps on her left side. c. The mother routinely uses cocaine. d. The mother drinks one caffeine drink a day.

C Cocaine use produces maternal hypertension. Hypertension in the mother reduces blood flow to the placenta and decreases the fetal oxygenation.

A primigravida is in the latent phase of labor and is at low risk for complications of labor. She asks the nurse if she may walk for a few minutes. The nurse is aware that this is (is not) possible because: a. Continuous fetal monitoring is required. b. Continuous monitoring of the contractions is necessary at this stage of labor. c. Intermittent auscultation of fetal heart rate is appropriate for her. d. There is no need to assess fetal heart rate at this early stage of labor.

C Continuous fetal and uterine monitoring are not necessary for the latent phase of labor in women who are at low risk for complications.

A client is being discharged, having been diagnosed with false labor. The nursing diagnosis for her is Deficient Knowledge: characteristics of true labor. An appropriate expected outcome for this diagnosis is that the: a. Client will return to the hospital when she is in true labor. b. Client will define true labor. c. Client will describe reasons for returning to the hospital for evaluation. d. Client will be able to determine false from true labor.

C The client may not be able to determine true from false labor; however, she should be made aware of what signs to look for that may indicate the need for evaluation.

Firm contractions that occur every 3 minutes and last 100 seconds may reduce fetal oxygen supply because they: a. Cause fetal bradycardia and reduce oxygen concentration. b. Activate the fetal sympathetic nervous system. c. Limit the time for oxygen exchange in the placenta. d. Suppress the normal variability of the fetal heart.

C The resting time between these contractions is about 80 seconds, which reduces the time available for exchange of oxygen and waste products in the placenta. This will reduce the fetal oxygen supply. The other choices can all be results of the decreased oxygen supply.

Which of the following findings meets the criteria of a reassuring FHR pattern? a. The FHR does not change as a result of fetal activity. b. The average baseline rate ranges between 90 and 110 bpm. c. Mild late deceleration patterns occur with some contractions. d. Variability averages between 6 and 25 bpm.

D Variability indicates a well-oxygenated fetus with a functioning autonomic nervous system. The FHR should accelerate with fetal movement. Baseline range for the FHR is from 110 to 160 bpm. Late deceleration patterns are never reassuring.

The technique of delaying pushing until the reflex urge to push occurs may be called _____________________.

Delayed pushing, laboring down, rest and descend, or passive pushing


Kaugnay na mga set ng pag-aaral

The Mexican-American War, The Mexican Cession, and the Treaty of Guadalupe-Hidalgo

View Set

Ch 11 - How Genes are Controlled

View Set

Six Sigma Test 11 (ASQ Handbook Part 4)

View Set

Development through the lifespan chapter 1

View Set

missed questions PrepU: Hematologic Function and

View Set